Download as pdf or txt
Download as pdf or txt
You are on page 1of 76

MBA CET 2024 Expected paper

MBA CET 2024


Expected paper

1 | P a g e For classes | Shortcut workshops | mocks | books Cetking –


09594441448 | 09930028086| 09820377380 | www.cetking.com
MBA CET 2024 Expected paper
1. Select the option in which the numbers are related in the same way as are the numbers of the following set. (24, 10, 392)
1. (26, 12, 369)
2. (27, 15, 480)
3. (29, 18, 242)
4. (21, 18, 242)
5. (29, 18, 234)

2. Four letter-clusters have been given, out of which three are alike in some manner and one is different. Select the letter-cluster
that is different.1. NPSV 2. QTWZ 3. XADG 4. BEHK 5. MDHE

3. Select the letter-cluster from among the given options that can replace the question mark (?) in the following series.
BOARD, BOARW, BOAIW, ?, BLZIW, YLZIW
1. BOZIW
2. BOAIW
3. BLZRW
4. BOZRW
5. BOZSW

4. In a certain code language, ‘AND’ is written as ‘C-LP-F’ and ‘NOR’ is coded as ‘P-MQ-T’. How will ‘BUT’ be written in that
language?
1. D-SW-U
2. C-SU-V
3. D-SW-V
4. C-TV-W
5. D-SW-W

5. Select the option that is related to the third word in the same way as the second word is related to the first word.
Awful : Nice : : Nomadic : ?
1. Homeless 2. Careless 3. Nocturnal 4. Settled 5. Traveler

6. Select the correct option that indicates the arrangement of the given words in the order in which they appear in an English
dictionary.
1. Success 2. Surreal 3. Succumb 4. Suction 5. Surrogate 6. Surprise
1. 1, 4, 3, 6, 2, 5
2. 1, 3, 6, 4, 2, 5
3. 1, 3, 4, 6, 5, 2
4. 1, 3, 4, 6, 2, 5
5. 3, 1, 4, 6, 2, 5

7. In a certain code language, 'SAFETY' is coded as '80' and 'EXPAND' is coded as '68'. How will 'GATHER' be coded in that
language?
1. 68 2. 60 3. 66 4. 64 5. 63

8. The following Venn diagram shows the number of families who have visited three different places (Srinagar, Shimla,
Gangtok). What is the number of families who have visited at least two of the places out of Srinagar, Shimla and Gangtok?

1. 46 2. 47 3. 48 4. 49 5. 50

2 | P a g e For classes | Shortcut workshops | mocks | books Cetking –


09594441448 | 09930028086| 09820377380 | www.cetking.com
MBA CET 2024 Expected paper
9. In a certain code language ‘Indian banks are associates’ is written as ‘za pn ka sh’, ‘SBI has asscociates banks’ is written as ‘pn
za bi ti’, ‘national and Indian institute’ is written as ‘na ha sh sn’ and ‘national has international banks’ is written as ‘bi na mn pn’.
Find the code for Indian National Bank
a. sh na pn
b. sh ha sn
c. sh za ka
d. ti pn na
e. na pn bi

10. Pointing to the photograph of a girl, Raghav said, “She is my father’s mother’s daughter’s only brother’s daughter”. How is
Raghav’s father related to that girl’s mother?
1. Father 2. Maternal uncle 3. Son 4. Husband 5. None of these

11. Study the following information and answer the given questions.
• D is daughter of N. E is wife of N.
• G is sister of D. C is married to G.
• N has no son. K is mother of E.
• Q is only daughter of C.
How N is related to K?
1. Brother-in-law
2. Cousin
3. Son in law
4. Sister
5. Brother

12. When number X multiplied to its next number, the product 1260 is obtained. When Number X is added to Number Y, a total of
106 is obtained. Find the value of Number Y. (Assume numbers to be positive)
1. 77 2. 61 3. 87 4. 71 5. None of these

13. Select the combination of letters that when sequentially placed in the blanks of the given series will complete the series. b _ c e
_kb_c_fkbb_ef_bbc_fk
1. b, f, b, e, c, e, k
2. b, e, b, f, c, k, e
3. b, f, b, e, c, k, e
4. b, f, b, c, e, k, e
5. None of these

14. Following are the conditions for selecting a candidate in an organization.


The candidate must –
(i) have passed class 12 exam with at least 60% marks.
(ii) have passed graduation degree in any discipline with at least 55% marks.
(iii) be not less than 21 years and not more than 30 years of age as on 1.7.2016.
(iv) possess a certificate/diploma/degree course in Computer Science.
In the case of candidate who fulfills all conditions except –
(a) at (ii) above but is a post-graduate, case may be referred to the Executive Director (ED).
(b) at (iv) above but has studied Computer Science as one of subjects in graduation, case may be referred to the Vice President (VP).

In question below, details of a candidate are provided followed by a question and 5 courses of action. Select the course of action that
applies to the person’s candidature. All these cases are given to you as on 1.11.2016.

Lakshay is a Commerce graduate with 57% marks. He had secured 73% marks class 12 examination. He has studied Computer
Science as one of the subjects in class 12. His date of birth is 22.9.1990.
A) if the case is to be referred to Executive Head.
B) if the case is to be referred to Vice President.
C) if the candidate is to be selected.
D) if the information is inadequate to take a decision.
E) if the candidate is not to be selected.

15. Select the option that is related to the third number in the same way as the second number is related to the first number. 21 :
445 : : 24 : ?
1. 495 2. 505 3. 523 4. 580 5. 555

16. Statements:

3 | P a g e For classes | Shortcut workshops | mocks | books Cetking –


09594441448 | 09930028086| 09820377380 | www.cetking.com
MBA CET 2024 Expected paper
All dancers are talented.
Some girls are dancers.

Conclusions:
I. Some girls are talented.
II. All talented are girls is a possibility
III. All girls can be talented

1. Only conclusion III follows


2. Both conclusions I and II follow
3. Only conclusion II follows
4. Only conclusion I follows
5. All follows

17. Four number-pairs have been given, out of which three are alike in some manner and one is different. Select the number-pair
that is different.
1. 5237 : 4003 2. 4327 : 2003
3. 3527 : 2293 4. 2347 : 1113
5. All pairs are same

Options for Ques 16 – 18 is given below


A.

B.

C.

D.

E.

18. Which of the following diagrams best depicts the relationship among Doctors, Lawyers and Professionals?
19. Which of the following diagrams best depicts the relationship among Water, Atmosphere and Hydrogen?
20. Which of the following diagrams best depicts the relationship among Book, Dictionary and Printer?

21. Statements:

4 | P a g e For classes | Shortcut workshops | mocks | books Cetking –


09594441448 | 09930028086| 09820377380 | www.cetking.com
MBA CET 2024 Expected paper
Some bowls are plates.
No plate is a glass.

Conclusions:
I. Some glasses are not bowl.
II. All glasses being bowl is a possibility.

1. Either conclusion I or II follows


2. Both conclusions I and II follow
3. Only conclusion II follows
4. Only conclusion I follows
5. None follows

G is the mother of F, who is the spouse of D. M is the daughter of D, who is the only brother of C. E is the son of G, who is married
to H. A is the niece of C, who has no sister and is unmarried. T is the father of D and has no daughter. V is the sister-in-law of F. G
has only two children. M is the granddaughter of O.

22. How is V related to M?


A Paternal aunt B Maternal aunt
C Sister D Either A or B
E None of these

23. How many female members are there in the family?


A 7 B 4C 5 D6 E Can't be determined

24. Find the odd one out?


AV BD CC DF EA

25. How is F's mother-in-law related to T?


A Sister B Father
C Wife D Brother
E None of these

26. Who among the following is the sibling of F?


AG BH CV DE E None of these

27. Statements:
All flowers are plants.
No plant is an stem.
All stems are fruits.

Conclusions:
I. All flowers being fruits is a possibility.
II. All stems being flower is a possibility.

1. Either conclusion I or II follows


2. Both conclusions I and II follow
3. Only conclusion II follows
4. Only conclusion I follows
5. None follows

28. Statements:
Some pens are keys
Some keys are locks
All locks are cards
No card is a paper

Conclusions:
I. No lock is paper.
II. Some cards are keys
III. Some keys are not paper.

1. Either conclusion I or II follows

5 | P a g e For classes | Shortcut workshops | mocks | books Cetking –


09594441448 | 09930028086| 09820377380 | www.cetking.com
MBA CET 2024 Expected paper
2. Both conclusions I and II follow
3. Only conclusion III follows
4. Only conclusion I follows
5. All follows

29. Statements:
Some apartments are flats.
Some flats are buildings.
All buildings are bungalows.
All bungalows are garden

Conclusions:
I. All apartments being building is possibility.
II. All bungalows are not buildings.
III. No flat is garden.

1. Either conclusion I or II follows


2. Both conclusions I and II follow
3. Only conclusion III follows
4. Only conclusion I follows
5. All follows

Virat goes to Supermarket store to buy fruits. He enters the market, turns to his right, walks 7m and takes oranges. Then, he turns to
his left, walks 3m and takes Peach. Again, he turns to his left, which is towards west, walks 7m and takes Papayas. Again, he turns to
his right and looks at guavas, which 8m away from him. He takes guavas and moves to his right, walks 7m and stops at the counter
for billing.

30. In which direction does Virat move for billing?


A North B East C West
D Can’t be determined
E None of these

31. In which direction is the Papaya stall located with respect to the Orange stall?
A North B South C Northwest D Southeast E None of these

32. What is the shortest distance between the Orange stall and the Billing Counter?
A 11 m B 10 m C 12 m D 7 m E None of these

33. How far is the Papaya stall from the starting point?
A 3 m B 7 m C 8 m D 6 m E None of these

34. In a certain code language, 6219 means ‘Sachin is a cricketer’ and 2646 means ‘He played from Mumbai’. Which of the
following is the code for ‘Mumbai is very famous’?
1. 7945 2. 6246 3. 6285 4. 2458 5. 5482

35. Select the option in which the words share the same relationship as that shared by the given pair of words.
Dentist : Hospital
1. Engineer : Design
2. Criminal : Police Station
3. Professor : University
4. Doctor : Treatment
5. None of these is correct

6 | P a g e For classes | Shortcut workshops | mocks | books Cetking –


09594441448 | 09930028086| 09820377380 | www.cetking.com
MBA CET 2024 Expected paper
The following pie-charts show the percentage distribution of the total employees of two Companies A and B
in different departments, and the table shows the ratio of Male to Female employees in all the departments of
Company A and B. The total number of employees working in Company A and B are 8000 and 7500
respectively.

1. What is the total number of Female employees in D5 of Company A and B together?


(a) 705 (b) 710 (c) 715 (d) 720 (e) 725

2. The total number of Female employees in D1 of Company B is approximately how much per cent more
than the number of Female employees in D1 of Company A?
(a) 5% (b) 7.5% (c) 15% (d) 22.5% (e) 30%

3. What is the difference between the total Male employees of Company A and the total Female employees
of Company B?
(a) 1230 (b) 1232 (c) 1234 (d) 1236 (e) 1238

4. The average number of Male employees in D1 and D2 of Company B is approximately what


percentage of the average number of Female employees in D5 and D6 of Company A? (a) 177.5% (b)
197.5% (c) 212.5% (d) 217.5% (e) 227.5%

5. The total number of Females working in Company A is approximately what percentage of total
employees of Company A?
(a) 42.12% (b) 43.48% (c) 44.24% (d) 45.64% (e) 46.86%

7 | P a g e For classes | Shortcut workshops | mocks | books Cetking –


09594441448 | 09930028086| 09820377380 | www.cetking.com
MBA CET 2024 Expected paper
Study the following information carefully and answer the questions given below:
In an institute there are 5600 students, the ratio of the number of girls to the number of boys is 3 : 4
respectively. All the students are enrolled in different programmes viz. SSC, Banking, CLAT and F–CAT.
20% of the total number of boys are enrolled in CLAT programme. The number of girls enrolled in Banking
is five–fourths of the number of boys taking the same.
One fourth of the total number of girls enrolled in F–CAT programme. The total number of students enrolled
in F–CAT programme is 1400. Two–fifths of the boys enrolled in SSC programme and the remaining boys
are enrolled in Banking programme. The girls enrolled in SSC are three fourth of the number of boys enrolled
in the same. The remaining girls are enrolled in CLAT programme.

6. What is the respective ratio of the number of boys enrolled in CLAT to the number of boys enrolled in
SSC?
1) 1 : 2 2) 1 : 3 3) 2 : 3 4) 4 : 5 5) 3 : 4

7. What is the number of girls enrolled in CLAT? 1) 210 2) 260 3) 280 4) 180 5) 240

8. What is the total number of students enrolled in Banking? 1) 1060 2) None of these 3) 1080 4) 1040 5)
1020

9. The number of girls enrolled in F–CAT is approximately what percent of the total number of students
enrolled in SSC? 1) 24.8% 2) 26.8% 3) 36.8% 4) 32.9% 5) 29.4%

10. The number of boys enrolled in CLAT and SSC together is approximately what percent more or less
than the number of girls enrolled in SSC?
1) 100% more 2) 100% less 3) 125% more 4) Other than given options 5) 75% less

8 | P a g e For classes | Shortcut workshops | mocks | books Cetking –


09594441448 | 09930028086| 09820377380 | www.cetking.com
MBA CET 2024 Expected paper
Directions Q11 to Q15- there are five travelling companies. And the following table gives the percentage
distribution of all customers according to their mode of transportation.

Q11. The total number of passenger travels through kali travels is 300, and the passenger travels by bus and
air for royal cruiser is equal to the passenger travel by all the modes other than Air from kali travels. Find the
ratio of traveler by Train from Royal cruiser to Bus traveler of kali travels. (a)138/225 (b)128/225 (c)124/123
(d)152/152 (e)None

Q12. If the total number of passenger is 500 from the royal cruise and same for Iana travels. Find the
difference of people travelling by Bus from these two travel agencies.
(a)44 (b)54 (c)45 (d)36 (e)None of the above

Q13. From the Kingfisher travels passenger travelling by train and ship is equal to the passenger travelling by
car. Then how much percentage of passenger is travelling by Bus?
(a)15 (b)16 (c)17 (d)18 (e) None of the above

Q14. Kingfisher passengers are 60% more than the Royal cruiser total passenger. Then the passenger
travelling by bus from kingfisher travels is how much percent more/less than that of royal travel travelling
through same mode of transport. (a)35 (b)37.37 (c)36.84 (d)47 (e)48.26

Q15. How many passengers are travelling by Nainital travels if car and bus traveler of Nanital travels is
equal to the Air traveler of Kingfisher travels.
(a) 500 (b) 550 (c) 560 (d) 450 (e) None of the above

Q16. To find out the share of Y out of Rs 1820, which of the following statements is/are sufficient/necessary?
A. The share of X is 1.8 times the combined share of Y and Z.
B. The share of Y is 3/11 of the combined share of X and Z.
C. The share of Z is 1/6of the combined share of X and Y.
(a) Statements A and B together are sufficient
(b) Statement A and C together are sufficient
(c) Statements B and C together are sufficient
(d) Either statement B alone or statements A and C together are sufficient
(e) None of these

Q17. A sum of money Rs 2550 is to be distributed among Knahiya, Varun and Rajneesh. What will be the
share of Varun?
A. Knhiya’s share is 1.5 times Varun’s share.
B. Rajneesh’s share is half the share of Knahiya and Varun together.
C. The share of Knahiya is Rs 340 more than Varun.
(a) Either A and B or A and C together are sufficient
(b) Only A and B (c) Only A and C (d) All are required
(e) Even all together are not sufficient

9 | P a g e For classes | Shortcut workshops | mocks | books Cetking –


09594441448 | 09930028086| 09820377380 | www.cetking.com
MBA CET 2024 Expected paper
Q18. To find the temperature on Monday which of the following information is sufficient?
A. The average temperature for Monday, Tuesday and Wednesday was 38° C.
B. The average temperature for Tuesday, Wednesday and Thursday was 43° C.
C. The temperature on Tuesday and Thursday was 45° C each.
(a) Only A and B are sufficient (b) Only B and C
(c) A and either B or C (d) C and either A or B
(e) All the three together are sufficient

Find the odd number in the following number series?

Q19. 550, 550, 549, 544, 515, 330


(a) 549 (b) 544 (c) 515 (d) 330 (e) 550

Q20. 450, 900, 500, 800, 640, 760


(a) 900 (b) 600 (c) 800 (d) 640 (e) 760

What will come in place of question mark (?) in the following series?
Q21. 23 25 30 40 57 ?
(a) 63 (b) 72 (c) 83 (d) 93 (e) 102

Q22. 13 24 92 546 4360 ?


(a) 47582 (b) 43590 (c) 54280 (d) 32590 (e) 63200

Directions: What will come in place of question mark(?) in the following questions?
Q23. (180 × 15 – 12 × 20) / (140 × 8 + 2 × 55) = ?
1) 1 2) 2 3) 3 4) 4 5) 5

Q24. (3/8) of 168 × 15 ÷ 5 + ? = 549 ÷ 9 + 235


1) 105 2) 160 3) 107 4) 180 5) 140

Compare Quantity x and Quantity y, using additional


information centered above the two quantities if
such information is given, and select one of the
following four answer choices:
A if x > y
B if x ≤ y
C if x ≥ y
D if x < y
E if x = y or relationship between x and y can't be
established

Q25. I. 11x + 64/x = 54


II. 12y2 + 40y + 17 = 0

Instructions for 1 and 2


A. Statement I is the cause and statement II is its effect
B. Statement II is the cause and statement I is its effect
C. Both the statements I and II are independent causes
D. Both the statements I and II are effects of independent causes
E. Both the statements I and II are effects of some common cause

1. Statements:

10 | P a g e For classes | Shortcut workshops | mocks | books Cetking –


09594441448 | 09930028086| 09820377380 | www.cetking.com
MBA CET 2024 Expected paper
I. Large number of people living in the low-lying areas has been evacuated during the last few days to safer places.
II. The Government has rushed in relief supplies to the people living in the affected areas.

2. Statements:
I. It is the aim of the city’s civic authority to get the air pollution reduced by 20% in the next two months.
II. The number of asthma cases in the city is constantly increasing.

3. Statement: Four districts in state A have been experiencing serve drought for the last three years resulting into exodus of people from
these districts.
Courses of Action:
I. The government should immediately start food for work program in the district to put a halt to the Exodus.
II. The government should make effort to provide drinking / potable water to these districts.
III. Government should start investing in projects for effective use of resources during draught time.
A. Only I and II follows
B. Only II follows
C. Either I or II follows
D. I and II and III follows
E. All I II and III follow

The cases of food poisoning due to consumption of liquor in rural areas are far greater than such cases in urban areas. Hence it can be
concluded that the liquor supplied in rural areas is of low quality than that supplied in urban areas.
(A) The prices of liquor are lower in rural areas than in urban areas.
(B) Percentage of people consuming liquor is more in rural areas than in urban areas.
(C) There are many unauthorized spurious liquor shops in the rural areas.
(D) Number of people suffering from food poisoning due to consumption of liquor as a percentage of people who consume liquor is almost
equal for both urban areas and rural areas.

4. Which of the above statements numbered (B), (C) and (D) if true will most weaken the given argument?
(1) Only D (2) Both B and D (3) Only C (4) Only B (5) None

5. Which of the above statements numbered (A), (B), (C) and if true will strengthen the given argument?
(1) Only A (2) Only B (3) Only C (4) Both A & D (5) A, C & D

6. The district administration has issued a circular to all the farmers under its jurisdiction advising them for not using pesticides
indiscriminately as it may pollute the ground water. Which of the following can be assumed from the given statement?
(1) People may stop using ground water if the farmers continue to use pesticides indiscriminately.
(2) The polluted ground water can cause health problems.
(3) Farmers may refrain from using pesticides indiscriminately.
(4) People in the district are majorly dependent on ground water for drinking use.
(5) None of these

7. Which of the following argument is correct?


Statement: Should those who receive dowry, despite the law prohibiting it, be punished?
I. Yes, those who violate the law, must be punished.
II. No, dowry system is firmly rooted in the society since time immemorial.
(1) Only I (2) Both I and II (3) Only II (4) I or II (5) None

8. According to Albert Einstein’s famous theory of relativity, time travel is theoretically possible. Assuming that time travel were to be
made possible through some technological breakthrough, it would be advantageous to send someone back in time to prevent the
assassination of Archduke Franz Ferdinand in 1914 and thus keep World War I from ever occurring. Which of the following inference
can be deduced from above argument?
(1) It is not possible to alter a significant current in world history merely by changing a single event.
(2) The technology necessary for time travel is likely to be developed in the near future.
(3) Franz Ferdinand was the most influential figure of 1914.
(4) The assassination of Franz Ferdinand was the crucial event that triggered the start of World War I.
(5) None of these

9. The airlines have requested all their passengers to check the status of flight operations before leaving their homes as heavy fog is
causing immense problems to normal flight operations. Which of the following inference can be made from the above statement?
(1) The majority of the air passengers may check the flight status before starting their journey to the airport.
(2) The Govt. may take serious objection to the notice issued by the airline company.
(3) Majority of the passengers may cancel their tickets and postpone their journey till the situation becomes normal.
(4) Both (1) and (2) (5) None of these

10. Which of the following assumption is correct?


Statement: The college administration has instructed all the students to stop using cell phone within the college premises.
Assumption:
I. The student may stop using cell phone in the college premises

11 | P a g e For classes | Shortcut workshops | mocks | books Cetking –


09594441448 | 09930028086| 09820377380 | www.cetking.com
MBA CET 2024 Expected paper
II. Some student may continue to use cell phone in the college premises.
(1) Only I (2) Both I and II (3) Only II (4) I or II (5) None

Select the phrase/connector out of three phrases/connectors given as (a), (b) and (c) which can be used in the
beginning (to start the sentence) to form a single sentence from the two or three statements given while
implying the same meaning as expressed in the statement sentences.

1. I. India has implemented an action plan formulated by the Centre’s Department of Animal Husbandry,
Dairying and Fisheries to deal with avian influenza.
II. India is a major agricultural nation with a large poultry industry.
a. Although India …………
b. As a ………....
c. Being a …………..

A Only a
B Only c
C Only b
D b and c
E All of the above

2. I. Banks in India have embraced technological change.


II. The onus is on them to integrate inter-generational legacy systems across branches, ATMs and online
banking networks into one seamless and secure whole.
a. On account of technological ………..
b. Besides the onus ……………
c. With banks in India …………….

A Only b
B Only c
C Only a
D b and c
E None of these

Directions: Select the phrase/connector out of three phrases/connectors given as (a), (b) and (c) which can be
used in the beginning (to start the sentence) to form a single sentence from the two or three statements given
while implying the same meaning as expressed in the statement sentences.

3. I. The long queues in the outpatient clinics at medical colleges and general hospitals are often the first
indicator of the beginning of the epidemic season in Kerala.
II. Usually, dengue cases begin to peak with the arrival of the southwest monsoon.
a. As long as the long queues ....
b. Though, usually, dengue cases ....
c. Surpassing the beginning of ....
A Only c
B Only b
C a and b
D b and c
E None of these

Directions: In the following questions two statements have been given with connectors given as options. You
have to select that option which could connect both the sentences and make them grammatically and
contextually correct.

12 | P a g e For classes | Shortcut workshops | mocks | books Cetking –


09594441448 | 09930028086| 09820377380 | www.cetking.com
MBA CET 2024 Expected paper
4. I. Higher interest rates make equity investments unattractive as it weighs on earnings growth.
II. Higher interest rates lead to tightening of earnings yield spread over bond yields.
A Not only, besides
B Even though, for
C In addition, least
D Similarly, Otherwise
E None of the above

Directions: In the following questions two statements have been given with connectors given as options. You
have to select that option which could connect both the sentences and make them grammatically and
contextually correct.

5. I. Instead of confiscating all properties belonging to a fugitive economic offender, a prudent step would be
to quantify the outstanding loan amount.
II. Only those properties must be confiscated that have been acquired through proceeds of crime.
A Also, too
B Presently, finally
C Accordingly, hence
D Nearby, adjacent to
E None of the above

A. Then two astronomers—the German, Johannes Kepler, and the Italian, Galileo Galilei—started publicly to
support the Copernican theory, despite the fact that the orbits it predicted did not quite match the ones
observed.
B. His idea was that the sun was stationary at the centre
and that the earth and the planets move in circular orbits around the sun.
C. A simple model was proposed in 1514 by a Polish priest, Nicholas Copernicus.
D. Nearly a century passed before this idea was taken seriously.

6. When put in correct order what will come FIRST Position


A. A B. B C. C D. D E. None of these

7. When put in correct order what will come SECOND Position


A. A B. B C. C D. D E. None of these

8. When put in correct order what will come THIRD Position


A. A B. B C. C D. D E. None of these

9. When put in correct order what will come FINAL Position


A. A B. B C. C D. D E. None of these

13 | P a g e For classes | Shortcut workshops | mocks | books Cetking –


09594441448 | 09930028086| 09820377380 | www.cetking.com
MBA CET 2024 Expected paper
10. Find the correct order for the following passage
A. People can get infected by handling reptiles and then touching their mouths or an open cut.
B. At first they look the perfect pets: exotic, quiet and tidy.
C. A study estimates that in 1995, there were as many as 6,700 reptile-caused salmonella infections.
D. But lizards and other pets can harbour a salmonella bacterium that makes people sick.
(a) BCAD (b) BDAC (c) ADCB (d) BDCA (e) CDBA

During the 2014 national and State elections, for the first time women’s safety and empowerment were topics
of debate, marking a significant shift in how gender concerns are viewed by the political class as well as by
voters in India. In the two years since, policy focus and public scrutiny on persistent gender inequality has grown
exponentially. In 2015, 194 member states, including India, adopted the Sustainable Development Goals.
Gender equality is one of the 17 goals to “transform our world”. This year, India ratified the Paris Agreement.
The direct link between empowering women and alleviating poverty, increasing productivity, and combating
climate change is well-recognised. However, the lack of targeted resources is often stated to be the biggest
reason behind the sluggish progress in furthering the gender agenda. Therefore, it is important that India’s
budget priorities reflect its commitment to invest in women and girls.

Last year, the World Economic Forum’s annual Global Gender Gap Report ranked India 87 in terms of gender
equality in economy, education, health, and political representation. Women’s declining labour participation,
under-representation in Parliament, skewed child sex ratio, and prevalent gender-based violence are recognised
challenges. To bridge these gaps, India formally adopted Gender Responsive Budgeting (GRB) in 2005. The
rationale behind GRB is that policy outcomes are not as gender-neutral as commonly believed, and can reinforce
or exacerbate exiting hierarchies. Hence, gender budgeting initiatives aim to integrate critical gender concerns
into fiscal policies and administration to address disparities.

Every annual budget since 2005 has included a statement that lists out two parts. There is Part A, which reflects
‘Women Specific Schemes’, namely, those which have 100 per cent allocation for women, and Part B, which
reflects ‘Pro Women Schemes’, namely, where at least 30 per cent of the allocation is for women. Over the
years, India has stood out for its implementation of gender budgeting, and with the Ministry of Finance (MoF)
playing the central role, it has managed to successfully institutionalise the concept at both the national and State
levels (16 States have embraced the exercise). Studies substantiate the positive link between GRB and improved
indicators for women. For instance, a recent International Monetary Fund study found that States that employ
GRB also show better female to male school enrolment ratios. Further, it was observed that GRB also has a
positive impact on infrastructure spending.

Despite the successes, better implementation and planning are needed to ensure that these policies percolate
right down to the last woman in the most remote parts of the country. In recent years, allocations have either
remained stagnant or have been on the decline. For instance, Budget 2016-17 was widely considered to be a
mixed bag for women. While the Ministry of Women and Child Development and National Commission for
Women saw nominal increases, the scheme meant for implementing the Domestic Violence Act did not receive
any allocation. Further, there was a decline in the number of ministries and departments that fall under GRB.
The budget also initiated the decentralisation of funding in GRB, thus shifting the onus for budgeting and
implementation from the Central Ministry to State counterparts. While this did empower the States to come up
with women-specific policies as per their respective challenges, the obvious downside was the risk that States
could choose to not prioritise gender in their budgeting. In this way, the intent of universalising the process, so
that it equally benefits women in all States, was lost in the pragmatism of the move and for it to be truly effective,
GRB must be viewed as an essential tool to tackle societal inequality that hinders progress instead of a symbolic
exercise for pleasing the emerging women constituency.

11. What does the statement “Budget 2016-17 was widely considered a mixed bag for women.” express?
A The decisions taken for women in the Budget 2016-17 were both positive and not so positive in nature.
B The government has announced various policies targeting women of India.
C Though the fund allocation has been raised yet a number of ministries were taken out.

14 | P a g e For classes | Shortcut workshops | mocks | books Cetking –


09594441448 | 09930028086| 09820377380 | www.cetking.com
MBA CET 2024 Expected paper
D A lot of policies that government has brought forward in the Budget seem to be giving mixed results.
E None of these

12. Which of the following statements is/are true in the context of the passage?
A In the List B under GRB at least 30% of the fund allocation is for women working in unorganised sector.
B In Budget 2016-17 there was an increase in ministries and departments that fall under GRB.
C The planning commission has been playing the Central role as far as implementation of GRB is concerned.
D Implementation of GRB had a positive impact only on one area – Infrastructure spending.
E All are false

13. Which of the following steps with reference to the passage was not supported by the author completely?
A Ratification of Paris Agreement by India
B Integration of critical gender concerns into financial policies through GRB.
C Institutionalisation of GRB at both the national and State levels.
D Decentralisation of funding in GRB
E None of these

14. Which one of these words is opposite to the word ‘exacerbate’ highlighted in the given passage?
A Illustrate
B Aggravate
C Justify
D Topple
E Ameliorate

15. Which one of these words is similar to the word ‘onus’ highlighted in the given passage?
A sufferings
B position
C responsibilities
D directions
E roles

16. According to the author, India’s marking a significant shift in the outlook of the political class and that
of the voters towards gender concerns is best illustrated by the fact that:
A India’s political class and the mass have become aware that a comprehensive growth can’t be achieved without
bringing gender equality to the system.
B The issues regarding women’s safety and empowerment have been discussed during the 2014 national and
State elections for the first time.
C The government has recently taken a lot of sincere steps to curb corruption that exists in the women
empowerment policy making body.
D Indian government has recently built and strengthened its targeted resources in furthering the gender agenda.
E None of these

17. Which of the following statements can be inferred from the passage?
A Gender is a key dimension for analysis of the impact of climate change.
B Requirement of funds in the Ministry of Women and Child Development has always been met.
C No concrete action has been taken towards women empowerment before 2005.
D India’s current rank in the World Economic Forum’s annual Global Gender Gap report is satisfactory to some
extent.
E None of these

Direction: Read the following passage carefully and answer the questions that follow.

15 | P a g e For classes | Shortcut workshops | mocks | books Cetking –


09594441448 | 09930028086| 09820377380 | www.cetking.com
MBA CET 2024 Expected paper
Start-ups are heaving a sigh of relief as the Finance Minister institutionalise measures to do away with the long
pending contentious issue of Angel Tax, which many of them had to cough up for raising Angel funding under
Section 56 of the Income Tax Act.

Also, special administrative arrangements will be made by the Central Board of Direct Taxes for pending
assessments of start-ups and redressal of their grievances. Start-ups have been assured that no inquiry or
verification in such cases can be carried out by the Assessing Officer without obtaining approval of his
supervisory officer. This provision will do away with much of the angst among start-ups who were subjected to
aggressive questioning by the Income Tax department.

A TV channel exclusively for start-ups under the Doordarshan boutique will be designed and executed by start-
ups themselves and will also serve as a platform for promoting them, discussing issues affecting their growth,
match making with VCs and for funding and tax planning, came as a pleasant surprise to many. Although it
remains to be seen how it will be executed on the ground.

The Budget sounded the bugle for fostering rural entrepreneurship for those depending on agriculture and
traditional industries by announcing the setting up of 80 Livelihood Business Incubators and 20 Technology
Business Incubators this fiscal with the aim to develop 75,000 skilled entrepreneurs in agro-rural industry
sectors. “The Government has tried to broad base and foster entrepreneurship beyond metros to Tier 2 and 3
cities which is a good sign. And doing away with Angel Tax is welcome” said Bhaskar Majumdar, Managing
Partner, Unicorn India Ventures.

18. Which among the following can be said regarding the sectoral thrust given by the government on
startups of the country?
A The government has made sure that the primary sector of the country gets the fair share of attention from the
start-up industry.
B The government is of the opinion that the startup industry should be profitable in the country first so that it
expands to other areas also.
C The government has decided to ensure that there is no friction between the sectors in the economy.
D The government has made sure that all the sectors in the economy get the required attention for growth and
development.
E None of the above

19. Which among the following is correct regarding the attitude of the government towards the startup
economy of the country?
A The economy of the country has been given a lifeline but there are a lot of questions to ponder over that.
B The economy of the country is mainly concerned about the startups of the nation though the government has
not done anything for them.
C The country is still reliant on big industrial houses so that they can pay taxes as much as possible.
D The government has understood the importance of the startups in the economy and is positive about the
development of this economy.
E None of the above

20. Which among the following is /are correct regarding the TV channel that has been announced by the
government in the Budget?
I. This new channel will be run by the government with the experts from the government organizations.
II. This new channel will be financed by the public sector banks with a fresh round of capital from the
government only.
III. This new channel will attempt at saving the taxes of the startup founders by the experts.
A Both II and III
B Both I and III
C Both I and II
D Only III

16 | P a g e For classes | Shortcut workshops | mocks | books Cetking –


09594441448 | 09930028086| 09820377380 | www.cetking.com
MBA CET 2024 Expected paper
E None of I, II and III

21. What is the opinion of the author regarding the TV channel that has been proposed to be set up by the
government?
A The author is excited about the new TV channel since it will bring a lot of interest in the sector.
B The author has nothing to say regarding the new TV channel that has been proposed in the Union Budget.
C The author is cautious in his reaction because he wants to see if the government is able to make it work
actually.
D The author is disappointed with this decision for the fact that the government should have done something
more worthwhile for the startups in India.
E None of the above

22. Which among the following is/are correct regarding the Angel Tax levied by the government on the
startups in India?
I. Angel Tax is levied on companies that raise capital from the angel investors only and not from others sources.
II. Angel Tax is not defined in the Income Tax legislation and the government had to implement it separately.
III. Angel Tax is not going to be levied on the companies that go out of business from this year onwards.
A Both I and II
B Only I
C Both II and III
D Both I and III
E All I, II and III

23. Which of the following word is similar in meaning as institutionalise?


A. Establish a company
B. Establish an activity
C. Establish a game
D. Establish a problem
E.None of these

24. What is the reason author mentions “Sounded the bugle”?


A. Start schemes
B. Stopped corruption
C. Help Entrepreneurs
D. Turn a blind eye
E.Stay put

25. Which of the following word is Antonym of Contentious?


A. Dormant
B. Disagreement
C. Important
D. Improvement
E.Relaxed

Eight persons – Ankit, Kavit, Manit, Vinit, Charit, Sumit, Ganit, and Ronit are sitting in a straight line (but not
necessarily in the same order) such that three of them are facing south and rest of them are facing north. Each
of them works in a different bank – SBI, UCO, PNB, BOB, SEBI, RBI, BOI, and NHB (but not necessarily in
same order).
Four persons sit between Ronit and Charit, who works in RBI. Sumit sits right of Manit, who works in BOI.
Ganit and Charit face opposite direction to each other. The one, who is sitting on the immediate right of Vinit,
works in NHB. Kavit sits third from one of the extreme end of the row. Ronit sits third to the right of Manit. The
one who works in PNB sits immediate right of the one who works in UCO. Manit doesn’t face north direction.

17 | P a g e For classes | Shortcut workshops | mocks | books Cetking –


09594441448 | 09930028086| 09820377380 | www.cetking.com
MBA CET 2024 Expected paper
Ganit is an immediate neighbor of Manit and Charit both. Vinit sits second to the right of Ganit. Manit sits on
the immediate left of Kavit. The one who works in SBI sits at an extreme end. The one who works in BOB sits
between Charit and Ankit. Sumit does not work in PNB.

1. Find the odd man out? Data Map


a) SBI b) UCO c) SEBI
d) BOB e) RBI

2. Sumit: Kavit : : Ronit : ____


a) Kavit b) Manit c) Charit
d) Vinit e) Sumit

3. How many people are sitting


between Ronit and Manit?
A One B Two C Three
D Four E Five

4. Who sits Fourth from Charit?


a) Ronit b) Manit c) Charit
d) Vinit e) Sumit

5. How many people are sitting


between Vinit and Manit?
A One B Two C Three
D Four E Five

Name

Bank

Direction

Name

Bank

Direction

18 | P a g e For classes | Shortcut workshops | mocks | books Cetking –


09594441448 | 09930028086| 09820377380 | www.cetking.com
MBA CET 2024 Expected paper
Six students namely, Aniruddh, Nishith, Darshit, Indrajit, Lakshit and Jayesh are studying in same school and
are from different places namely Chennai, Madurai, Tirunelveli, Trichy, Coimbatore and Nagercoil but not
necessarily in the same order. All of them got different ranks in annual exam from 1 to 6 but not necessarily
in the same order. All of them purchase dresses for Diwali festival and the price ranges from Rs. 200 to
Rs.450 in consecutive multiples of Rs.50 but not necessarily in the same order.
Nishith got 2nd rank in annual exam. Aniruddh is from Nagercoil. The sum of the price of the dresses
purchased by Aniruddh and Indrajit is equal to the sum of the price of dresses purchased by Nishith and
Lakshit. The price of the dress purchased by Indrajit is higher than the price of the dress purchased by
Nishith and Lakshit individually. The sum of the price of dresses purchased by Aniruddh and Indrajit is
Rs.600. The student who purchased dress for highest amount is from Trichy. The student who got 1st rank in
annual exam is from Madurai. Neither Lakshit nor the one who purchased dress for Rs.350 is from Madurai.
The student who purchased dress for highest amount gets 4th rank in annual exam. The student who got
5th rank purchased dress for higher amount than the student who got 6th rank and 3rd rank individually. The
student who got 6th rank in annual exam is from Coimbatore. The student who got 5th rank purchased dress
for lesser amount than the one who is from Madurai. The student who got 5th rank is not from Chennai or
Nagarcoil. The one who is from Tirunelveli did not purchase dress for Rs.350. Darshit is not from Tirunelveli.

6. How much amount is spent by the student, who is from Data Map
Tirunelveli for purchasing dress?
A Rs.200 B Rs.250 C Rs.300 D Rs.400
E Can't be determined

7. Nishith belongs to which among the following places?


A Nagercoil B Chennai C Coimbatore
D Tirunelveli E Trichy

8. Which of the following ranks was obtained by the


student, who purchased the dress for the least amount?
A 5th B 6th C 2nd D 3rd E Can't be determined

9. How many students purchased dress for an amount


higher than the one who got 6th rank in annual exam?
A One B Two C Three D Four E Five

10. Who among the following students belongs to


Coimbatore?
A Lakshit B Indrajit C Nishith D Jayesh
E Can't be determined

Rank 1 2 3 4 5 6

Student

Place

Dress (Rs)

Student

Place

Dress (Rs)

19 | P a g e For classes | Shortcut workshops | mocks | books Cetking –


09594441448 | 09930028086| 09820377380 | www.cetking.com
MBA CET 2024 Expected paper
Eight boxes P to W are placed in a rack one above another, but not necessarily in the same order. Each of
the boxes has different number of balls. Lowermost rack is numbered as one and above is two and so on.
Box R is placed ________ (I) boxes above the box, which 89 balls. Two boxes are placed between Box P
and Box U. Box ______ (II), which has 44 balls, is placed immediately above the Box T. Sum of balls in Box
P and Box Q is 80. Box W has 93 balls and it is placed exactly between Box Q and Box V. At most two boxes
are placed above Box T. Number of boxes placed above the Box T is same as the number of boxes placed
below Box V. Box P, which has _______ (III), balls is placed above Box V. Four boxes are placed between
the boxes, which have 67 balls and 59 balls. Box R has 72 balls and placed in ________ (IV) rack. Two
boxes are placed between the boxes, which has 59 and 81 balls. The box, which has 67 balls, is placed
immediately below the Box P. Box Q, which has _______ (V) balls, is placed in four racks above the Box V.

11. Fill the blank I? Data Map


1. 1 2. 2 3. 3 4. 4 5. 5

12. Fill the blank II?


1. P 2. T 3. U 4. W 5. S

13. Fill the blank III?


1. 67 2. 36 3. 81 4. 59 5. 89

14. Fill the blank IV?


1. 1 2. 2 3. 3 4. 4 5. 5

15. Fill the blank V?


1. 67 2. 36 3. 81 4. 59 5. 89

8 7 6 5 4 3 2 1

20 | P a g e For classes | Shortcut workshops | mocks | books Cetking –


09594441448 | 09930028086| 09820377380 | www.cetking.com
MBA CET 2024 Expected paper
A, B, C, D, E, F, G and H are eight friends. Travelling in three different cars, viz X, Y and Z, with at least two
in one car to three different places, viz. Delhi, Chandigarh and Agra. There is at least one female member in
each car. D is travelling with G to Delhi but not in car Y. A is travelling with only H in car Z but not to
Chandigarh. C is not travelling with either D or E. F and D are studying in the same only girls’ college. H, B
and G are studying in the same only boys’ college.

16. Which of the following is odd man out?


1. A 2. B 3. C 4. D 5. H Name Gender Place Car
17. B is in which Car?
1. X 2. Y 3. Z 4. Y or Z
5. Cannot be determined

18. Determine the Gender of C?


1. Girl 2. Boy 3. Trans 4. Mostly Girl
5. Cannot be determined

19. Which of following Combination is correct?


1. C is a girl going in Y 2. B is boy in car Delhi
3. E is a girl going to Delhi 4. F is a girl in car X
5. None of these

20. Determine the Gender of E?


1. Girl 2. Boy 3. Mostly Boy 4. Mostly Girl
5. Cannot be determined

21 | P a g e For classes | Shortcut workshops | mocks | books Cetking –


09594441448 | 09930028086| 09820377380 | www.cetking.com
MBA CET 2024 Expected paper
Uncertain no. of family members with 3 couples of a Cetking student are sitting around a circular table but not
facing centre. Only two people sit between T and W. T faces the centre. X sits second to the right of T. W is
the wife of S. No females is an immediate neighbor of W. U is not an immediate neighbor of T. U is the
daughter of W. Both the immediate neighbors of U face the centre. Only three people sit between S and U’s
brother. X is not the brother or husband of U. S, U and U’s brother none of them is an immediate neighbor of
X. V sits third from Y’s husband. Z, the wife of T, sits to the immediate left of V. Both Y and S face a direction
opposite to that of U (i.e if U faces the centre then both Y and S face outside and vice-versa). U’s husband
sits second to the left of Y. T’s father sits to the immediate right of W. T sits second to the right of S’s father.
Both the immediate neighbors of X are females. Y is grandmother of T.
Data Map
21. Who is Y is husband?
1. Y 2. T 3. U 4. W
5. None of these

22. Neighbors of X are?


1. S,W 2. T,U 3. U,S 4. W,Y
5. None of these

23. Who is grandfather of T?


1. Y 2. T 3. U 4. W
5. None of these

24. Who is brother of U?


1. Y 2. T 3. U 4. W
5. None of these

25. Who is sitting opposite of U?


1. Y 2. T 3. U 4. W
5. None of these

Family Tree

Name

Relation

Direction

Name

Relation

Direction

22 | P a g e For classes | Shortcut workshops | mocks | books Cetking –


09594441448 | 09930028086| 09820377380 | www.cetking.com
MBA CET 2024 Expected paper
Mercedes, Swift, Santro, Accord, Innova, Polo, Punto, Figo, Civic, City, Ferrari and Landrover
• Mercedes being a big car must be parked at one of the extreme ends. Mercedes is parked second to the
left of Santro. Santro faces Figo.
• Punto and Innova are parked immediately next to each other, Punto being parked left of innova. Neither is
parked next to Figo or Santro.
• Civic is parked in such a manner that its driver when seated in the parked car would face South. Civic is
parked third to the left of Polo.
• City is parked in such a manner that it faces Ferrari.
Ferrari is parked second to the left of Landrover. Landrover faces North and is not parked next to Figo.
• Swift is parked third to the right of Ferrari and faces Innova.

26. Which car is parked in the right end Data Map facing
north?
1. Mercedes 2. Polo 3. Punto
4. Accord 5. None of these

27. Which car is opposite Mercedes


1. Ferrari 2. Polo 3. Punto
4. Accord 5. None of these

28. Which car is diagonally opp Accord?


1. Ferrari 2. Polo 3. Punto
4. Accord 5. None of these

29. Which car is two places away Punto?


1. Ferrari 2. Polo 3. Punto
4. Figo 5. None of these

30. Polo : Ferrari :: City : ?


1. Mercedes 2. Polo 3. Punto
4. Santro 5. None of these

R↓L

L↑R

23 | P a g e For classes | Shortcut workshops | mocks | books Cetking –


09594441448 | 09930028086| 09820377380 | www.cetking.com
MBA CET 2024 Expected paper
1. A man is 24 years older than his son. In two years, his age will be twice the age of his son. The present age
of his son is:
A. 14 years
B. 18 years
C. 20 years
D. 22 years
E. 25 years

2. The angle between the minute hand and the hour hand of a clock when the time is 8.30, is:
A. 80°
B. 75°
C. 60°
D. 105°
E. 50

3. A, B and C jointly thought of engaging themselves in a business venture. It was agreed that A would invest
Rs. 6500 for 6 months, B, Rs. 8400 for 5 months and C, Rs. 10,000 for 3 months. A wants to be the working
member for which, he was to receive 5% of the profits. The profit earned was Rs. 7400. Calculate the share of
B in the profit.
A. Rs. 1900
B. Rs. 2660
C. Rs. 2800
D. Rs. 2840
E. Rs. 2940

4. The difference between simple and compound interests compounded annually on a certain sum of money
for 2 years at 4% per annum is Re. 1. The sum (in Rs.) is:
A. 625
B. 630
C. 640
D. 650
E. 660

5. A and B can do a piece of work in 30 days, while B and C can do the same work in 24 days and C and A in
20 days. They all work together for 10 days when B and C leave. How many days more will A take to finish
the work?
A. 18 days
B. 24 days
C. 30 days
D. 36 days
E. 40 days

6. The ratio between the speeds of two trains is 7 : 8. If the second train runs 400 km in 4 hours, then the
speed of the first train is:
A. 70 km/hr
B. 75 km/hr
C. 84 km/hr
D. 87.5 km/hr
E. 90 km/hr

7. In how many different ways can the letters of the word 'LEADING' be arranged in such a way that the
vowels always come together?

24 | P a g e For classes | Shortcut workshops | mocks | books Cetking –


09594441448 | 09930028086| 09820377380 | www.cetking.com
MBA CET 2024 Expected paper
A. 360
B. 480
C. 720
D. 5040
E. None of these

8. The difference between the length and breadth of a rectangle is 23 m. If its perimeter is 206 m, then its area
is:
A. 2530 m2
B. 2420 m2
C. 2480 m2
D. 2520 m2
D. 2560 m2

9. The difference between a two-digit number and the number obtained by interchanging the positions of its
digits is 36. What is the difference between the two digits of that number?
A. 3
B. 4
C. 9
D. Cannot be determined
E. None of these

10. A sum of money is to be distributed among A, B, C, D in the proportion of 5 : 2 : 4 : 3. If C gets Rs.


1000 more than D, what is B's share?
A. Rs. 500
B. Rs. 1500
C. Rs. 2000
D. None of these
E. Cannot be determined

11. A tank is filled by three pipes with uniform flow. The first two pipes operating simultaneously fill the
tank in the same time during which the tank is filled by the third pipe alone. The second pipe fills the tank 5
hours faster than the first pipe and 4 hours slower than the third pipe. The time required by the first pipe is:
A. 5 hours
B. 10 hours
C. 15 hours
D. 30 hours
E. 25 hours

12. A milk vendor has 2 cans of milk. The first contains 25% water and the rest milk. The second contains
50% water. How much milk should he mix from each of the containers so as to get 12 litres of milk such that
the ratio of water to milk is 3 : 5?
A. 6 litres, 8 litres
B. 6 litres, 6 litres
C. 5 litres, 7 litres
D. 6 litres, 5 litres
E. 7 litres, 6 litres

13. At a game of billiards, A can give B 15 points in 60 and A can give C to 20 points in 60. How many
points can B give C in a game of 90?
A. 30 points
B. 20 points
C. 10 points

25 | P a g e For classes | Shortcut workshops | mocks | books Cetking –


09594441448 | 09930028086| 09820377380 | www.cetking.com
MBA CET 2024 Expected paper
D. 12 points
E. 15 points

14. What is the probability of getting a sum 9 from two throws of a dice?
A. 3/6
B. 1/6
C. 3/9
D. 7/9
E. 1/9

15. A bag contains 4 white, 5 red and 6 blue balls. Three balls are drawn at random from the bag. The
probability that all of them are red, is:
A. 3/61
B. 2/61
C. 2/91
D. 7/91
E. 1/91

16. Which of the following statements is not correct?


A. log10 10 = 1

B. log (2 + 3) = log (2 x 3)

C. log10 1 = 0

D. log (1 + 2 + 3) = log 1 + log 2 + log 3


E. All are correct

17. A man's speed with the current is 15 km/hr and the speed of the current is 2.5 km/hr. The man's speed
against the current is:
A. 8.5 km/hr
B. 9 km/hr
C. 10 km/hr
D. 12.5 km/hr
E. 15 km/hr

18. A boat covers a certain distance downstream in 1 hour, while it comes back in 1 hours. If the speed of
the stream be 3 kmph, what is the speed of the boat in still water?
A. 12 kmph
B. 13 kmph
C. 14 kmph
D. 15 kmph
E. None of these

19. If a quarter kg of potato costs 60 paise, how many paise will 200 gm cost?
A. 48 paise
B. 54 paise
C. 56 paise
D. 72 paise
E. 40 paise.

20. A wheel that has 6 cogs is meshed with a larger wheel of 14 cogs. When the smaller wheel has made
21 revolutions, then the number of revolutions mad by the larger wheel is:

26 | P a g e For classes | Shortcut workshops | mocks | books Cetking –


09594441448 | 09930028086| 09820377380 | www.cetking.com
MBA CET 2024 Expected paper
A. 4
B. 9
C. 12
D. 49
E. 20

21. Find the value of x?

A. 2
B. 4
C. 3
D. 8
E. 5

22. Find the value of x?


If 3(x - y) = 27 and 3(x + y) = 243
A. 2
B. 4
C. 3
D. 8
E. 5

23. Let N be the greatest number that will divide 1305, 4665 and 6905, leaving the same remainder in each
case. Then sum of the digits in N is:
A. 4
B. 5
C. 6
D. 8
E. 7

24. The captain of a cricket team of 11 members is 26 years old and the wicket keeper is 3 years older. If
the ages of these two are excluded, the average age of the remaining players is one year less than the average
age of the whole team. What is the average age of the team?
A. 23 years
B. 24 years
C. 25 years
D. 26 years
E. 27 years

25. It was Sunday on Jan 1, 2006. What was the day of the week Jan 1, 2010?
A. Sunday
B. Saturday
C. Friday
D. Wednesday
E. None of these

27 | P a g e For classes | Shortcut workshops | mocks | books Cetking –


09594441448 | 09930028086| 09820377380 | www.cetking.com
MBA CET 2024 Expected paper
1. Find the odd man out of the following questions.

4. If the dice is resting on the side with three dots, what will be number of dots on the top?

1. 1 2. 2 3. 3 4. 4 5. 5 or 1

5. Find the number opp of 6?

1. 1 2. 2 3. 3 4. 4 5. 5

6. Select the figure which satisfies the same conditions of placement of the dots as in Figure-X.

28 | P a g e For classes | Shortcut workshops | mocks | books Cetking –


09594441448 | 09930028086| 09820377380 | www.cetking.com
MBA CET 2024 Expected paper

1. 1 2. 2 3. 3 4. 4 5. None of these

7. Find out from amongst the four alternatives as to how the pattern would appear when the transparent sheet is folded at the dotted
line.

(X) (1) (2) (3) (4)


1. 1 2. 2 3. 3 4. 4 5. None of these

8. Identify the figure that completes the pattern.

(X) (1) (2) (3) (4)


1. 1 2. 2 3. 3 4. 4 5. None of these

9. Choose the alternative which is closely resembles the water-image of the given combination.

29 | P a g e For classes | Shortcut workshops | mocks | books Cetking –


09594441448 | 09930028086| 09820377380 | www.cetking.com
MBA CET 2024 Expected paper
10. Find the number of triangles in the given figure.

1. 16 2. 18 3. 19 4. 17 5. 20

11. What is the number of straight lines and the number of triangles in the given figure.

A. 10 straight lines and 34 triangles


B. 9 straight lines and 34 triangles
C. 9 straight lines and 36 triangles
D. 10 straight lines and 36 triangles
E. 9 straight lines and 37 triangles

12. Select a suitable figure from the Answer Figures that would replace the question mark (?).

13. Select a suitable figure from the Answer Figures that would replace the question mark (?).

30 | P a g e For classes | Shortcut workshops | mocks | books Cetking –


09594441448 | 09930028086| 09820377380 | www.cetking.com
MBA CET 2024 Expected paper
14. Select the alternative which represents three out of the five alternative figures which when fitted into each other
would form a complete square.

1. 124 2. 123 3. 245 4. 342 5. 135

15. Select the alternative which represents three out of the five alternative figures which when fitted into each other
would form a complete square.

1. 124 2. 123 3. 145 4. 342 5. 135

16. Group the given figures into three classes using each figure only once.

A. 1,4,7 ; 2,5,8 ; 3,6,9


B. 1,4,7 ; 2,5,9 ; 3,6,7
C. 1,3,4 ; 2,5,8 ; 6,7,9
D. 1,2,3 ; 4,5,6 ; 7,8,9
E. None of these

17. Choose a figure which would most closely resemble the unfolded form of Figure (Z).

31 | P a g e For classes | Shortcut workshops | mocks | books Cetking –


09594441448 | 09930028086| 09820377380 | www.cetking.com
MBA CET 2024 Expected paper
18. Find the next time in the series below

19. Find the next time in the series below

20. Select a suitable figure from the four alternatives that would complete the figure matrix.

1. 1 2. 2 3. 3 4. 4 5. None of these

32 | P a g e For classes | Shortcut workshops | mocks | books Cetking –


09594441448 | 09930028086| 09820377380 | www.cetking.com
MBA CET 2024 Expected paper
21. Find out the alternative figure which contains figure (X) as its part.

(X) (1) (2) (3) (4)


1. 1 2. 2 3. 3 4. 4 5. None of these

22. Choose the set of figures which follows the given rule.
Rule: Any figure can be traced by a single unbroken line without retracting.

1. 1 2. 2 3. 3 4. 4 5. None of these

23. Find the next in the series

33 | P a g e For classes | Shortcut workshops | mocks | books Cetking –


09594441448 | 09930028086| 09820377380 | www.cetking.com
MBA CET 2024 Expected paper
24. Find the next in the series

25. Find the next in the series

34 | P a g e For classes | Shortcut workshops | mocks | books Cetking –


09594441448 | 09930028086| 09820377380 | www.cetking.com
MBA CET 2024 Expected paper
Directions for the next three questions

The phrases given in bold are correct. Which of the phrases (a), (b), (c) and (d) given below should replace the
phrase NOT given in bold to make the sentence grammatically meaningful and correct. If the sentence is
correct as it is and there is no correction required (e) as the answer.

1. As part of the Goods and Services Tax (GST) reforms, a new levy called the GST Compensation Cess have
been introduced to make good apprehended losses to States in the first five years of GST implementation.

(a) Cess had been introduced


(b) Cess has been introduced
(c) for making good apprehending
(d) apprehend losses to States in the
(e)No correction required

2. A legal personality is usually defined as a subject vested with rights and duties. However, within the
parameters of law, it have never been confined to human beings and has even included idols and companies.

(a) The legal personality is


(b) However, within
(c) it has never been confined
(d) confined to human beings
(e)No correction required

Which of the phrases (a), (b), (c) and (d) given below
should replace the phrase given in bold in the following sentences to make the sentence
grammatically meaningful and correct. If the sentence
is correct as it is and there is no correction required
(e) as the answer.

3. The biggest challenge India faces have that the


groups perceived to be disadvantage consist of very
larger segment of Indian society, while public policies
are highly limited in scope.
(A) faces is that the
(B) faces are that the
(C) a very large segment
(D) a very larger segment

(a) A and D are correct


(b) B and D are correct
(c) A and B are correct
(d) A and C are correct
(e) No correction required

Which of phrases given below each sentence should replace the phrase given in sentence to make the
grammatically correct? If the sentence is correct as it is, mark 'E' as the answer.

4. Because of his mastery in this field, his suggestions are wide accepted.
A. are widely accepted
B. widely acceptance
C. have widely accepted
D. have been wide accepted

35 | P a g e For classes | Shortcut workshops | mocks | books Cetking –


09594441448 | 09930028086| 09820377380 | www.cetking.com
MBA CET 2024 Expected paper
E. No correction required

5. They felt humiliated because they realised that they had cheated.
A. have been cheated
B. had been cheated
C. had been cheating
D. were to be cheated
E. No correction required

6. We don't know how did the thief made an escape.

A. how the thief did make


B. how the thief does make
C. how the thief made
D. how was the thief made
E. No correction required

7. Their earnings are such that they find it difficult to make both ends to meet.
A. to makings both ends meet
B. to make both ends for meeting
C. to make both ends meet
D. for making both ends to meet
E. No correction required

8. The logic of Berlin wall already had been undermined but when the news came through that the wall itself
had been opened I jumped into a car.
A. had been undetermined already
B. had already been undetermined
C. had been already undetermined
D. have been undermined already
E. No improvement

9. Which of the following sentence is grammatically correct?


A. The president told an address from the White House that US intelligence officials tracked al-Zawahiri to a
home in downtown Kabul where he was hiding out with his family.
B. The president said in an evening address from the White House that US intelligence officials tracked al-
Zawahiri to a home in downtown Kabul where he was hid with his family.
C. The president said on an evening address from the White House that US intelligence officials tracked al-
Zawahiri to a home in downtown Kabul where he was hiding out with his family.
D. The president said in an evening address from the White House that US intelligence officials tracked al-
Zawahiri to a home on downtown Kabul where he was hiding out with his family.
E. All are incorrect

10. Which of the following sentence is grammatically correct?


1. The United States is saying that Russia was using Ukraine's biggest nuclear power plant as a "nuclear
shield" by stationing troops there.
2. The United States said Russia was using Ukraine's biggest nuclear power plant as a "nuclear shield" by
capturing troops there.
3. The United States said Russia was utilising Ukraine's biggest nuclear power plant as a "nuclear shield" by
stationing troops there.
4. The United States told The Russia was using Ukraine's biggest nuclear power plant as a "nuclear shield" by
stationing troops there.

36 | P a g e For classes | Shortcut workshops | mocks | books Cetking –


09594441448 | 09930028086| 09820377380 | www.cetking.com
MBA CET 2024 Expected paper
5. The United States said Russia was using Ukraine's biggest nuclear power plant as a "nuclear shield" by
stationing troops there.

11. Find the correctly spelt words.


A. Bachhanalian
B. Bacchaanalian
C. Baachanalian
D. Bacchanalian
E. All are incorrect

12. Find the correctly spelt words.


A. Inoculation
B. Innoculation
C. Inocculation
D. Inocullation
E. All are Incorrect

13. Find the correctly spelt words.


A. Lackadaisicle
B. Lackdaisical
C. Lackadisical
D. Lackadaisical
E. All are Incorrect

14. Find the correctly spelt words.


A. Eflorescence
B. Efllorescence
C. Efflorescence
D. Efflorascence
E. All are Incorrect

15. Find the correctly spelt words.


A. Judicious
B. Cancious
C. Dilicous
D. Gracous
E. All are Incorrect

Some proverbs/idioms are given below together with their meanings. Choose the correct meaning of
proverb/idiom, If there is no correct meaning given, E (i.e.) 'None of these' will be the answer.

16. To leave someone in the lurch

A. To come to compromise with someone


B. Constant source of annoyance to someone
C. To put someone at ease
D. To desert someone in his difficulties
E. None of these

17. To play second fiddle

A. To be happy, cheerful and healthy


B. To reduce importance of one's senior

37 | P a g e For classes | Shortcut workshops | mocks | books Cetking –


09594441448 | 09930028086| 09820377380 | www.cetking.com
MBA CET 2024 Expected paper
C. To support the role and view of another person
D. To do back seat driving
E. None of these

In the earlier days, some long distance trains were ...(1)... more number of compartments, thus making the
train ...(2)... than even the length of platforms. Therefore, the last compartment usually ...(3)... outside the
platform. Once the person travelling in the last compartment of such a train could not ...(4)... tea, coffee,
snacks or water as he failed to get on the platform. He remained hungry and thirsty through out his journey.

18. Solve (1) as per the direction given above


A. attaching
B. shunting
C. travelling
D. manufacturing
E. having

19. Solve (2) as per the direction given above


A. bigger
B. longer
C. heavier
D. crowded
E. shorter

20. Solve (3) as per the direction given above


A. crowded
B. vacated
C. halted
D. derailed
E. collapsed

21. Solve (4) as per the direction given above


A. get
B. offer
C. eat
D. drink
E. sell

22. In the following questions choose the word which is the exact OPPOSITE of the given words.
SUBSERVIENT
A. Aggressive
B. Straightforward
C. Dignified
D. Supercilious
E. Elementary

23. In the following questions choose the word which is the exact OPPOSITE of the given words.
EXTRICATE
A. Manifest
B. Palpable
C. Release
D. Entangle
E. Confused

38 | P a g e For classes | Shortcut workshops | mocks | books Cetking –


09594441448 | 09930028086| 09820377380 | www.cetking.com
MBA CET 2024 Expected paper
24. In the following the questions choose the word which best expresses the meaning of the given word.
AUGUST
A. Common
B. Ridiculous
C. Dignified
D. Petty
E. Elementry

25. In the following the questions choose the word which best expresses the meaning of the given word.
CANTANKEROUS
A. Quarrelsome
B. Rash
C. Disrespectful
D. Noisy
E. Dormant

Grammar Grammar 10
Vocab Spelling 5
Idioms Idioms 2
Vocab Vocab Anto 4
Syno
Vocab Cloze / FIB 4

26. A man is 24 years older than his son. In two years, his age will be twice the age of his son. The present
age of his son is:
A. 14 years
B. 18 years
C. 20 years
D. 22 years
E. 25 years

27. The angle between the minute hand and the hour hand of a clock when the time is 8.30, is:
A. 80°
B. 75°
C. 60°
D. 105°
E. 50

28. A, B and C jointly thought of engaging themselves in a business venture. It was agreed that A would
invest Rs. 6500 for 6 months, B, Rs. 8400 for 5 months and C, Rs. 10,000 for 3 months. A wants to be the
working member for which, he was to receive 5% of the profits. The profit earned was Rs. 7400. Calculate the
share of B in the profit.
A. Rs. 1900
B. Rs. 2660
C. Rs. 2800
D. Rs. 2840
E. Rs. 2940

29. The difference between simple and compound interests compounded annually on a certain sum of
money for 2 years at 4% per annum is Re. 1. The sum (in Rs.) is:
A. 625
B. 630

39 | P a g e For classes | Shortcut workshops | mocks | books Cetking –


09594441448 | 09930028086| 09820377380 | www.cetking.com
MBA CET 2024 Expected paper
C. 640
D. 650
E. 660

30. A and B can do a piece of work in 30 days, while B and C can do the same work in 24 days and C and
A in 20 days. They all work together for 10 days when B and C leave. How many days more will A take to
finish the work?
A. 18 days
B. 24 days
C. 30 days
D. 36 days
E. 40 days

31. The ratio between the speeds of two trains is 7 : 8. If the second train runs 400 km in 4 hours, then the
speed of the first train is:
A. 70 km/hr
B. 75 km/hr
C. 84 km/hr
D. 87.5 km/hr
E. 90 km/hr

32. In how many different ways can the letters of the word 'LEADING' be arranged in such a way that the
vowels always come together?

A. 360
B. 480
C. 720
D. 5040
E. None of these

33. The difference between the length and breadth of a rectangle is 23 m. If its perimeter is 206 m, then its
area is:
A. 2530 m2
B. 2420 m2
C. 2480 m2
D. 2520 m2
D. 2560 m2

34. The difference between a two-digit number and the number obtained by interchanging the positions of
its digits is 36. What is the difference between the two digits of that number?
A. 3
B. 4
C. 9
D. Cannot be determined
E. None of these

35. A sum of money is to be distributed among A, B, C, D in the proportion of 5 : 2 : 4 : 3. If C gets Rs.


1000 more than D, what is B's share?
A. Rs. 500
B. Rs. 1500
C. Rs. 2000
D. None of these
E. Cannot be determined

40 | P a g e For classes | Shortcut workshops | mocks | books Cetking –


09594441448 | 09930028086| 09820377380 | www.cetking.com
MBA CET 2024 Expected paper
36. A tank is filled by three pipes with uniform flow. The first two pipes operating simultaneously fill the
tank in the same time during which the tank is filled by the third pipe alone. The second pipe fills the tank 5
hours faster than the first pipe and 4 hours slower than the third pipe. The time required by the first pipe is:
A. 5 hours
B. 10 hours
C. 15 hours
D. 30 hours
E. 25 hours

37. A milk vendor has 2 cans of milk. The first contains 25% water and the rest milk. The second contains
50% water. How much milk should he mix from each of the containers so as to get 12 litres of milk such that
the ratio of water to milk is 3 : 5?
A. 6 litres, 8 litres
B. 6 litres, 6 litres
C. 5 litres, 7 litres
D. 6 litres, 5 litres
E. 7 litres, 6 litres

38. At a game of billiards, A can give B 15 points in 60 and A can give C to 20 points in 60. How many
points can B give C in a game of 90?
A. 30 points
B. 20 points
C. 10 points
D. 12 points
E. 15 points

39. What is the probability of getting a sum 9 from two throws of a dice?
A. 3/6
B. 1/6
C. 3/9
D. 7/9
E. 1/9

40. A bag contains 4 white, 5 red and 6 blue balls. Three balls are drawn at random from the bag. The
probability that all of them are red, is:
A. 3/61
B. 2/61
C. 2/91
D. 7/91
E. 1/91

41. Which of the following statements is not correct?


A. log10 10 = 1

B. log (2 + 3) = log (2 x 3)

C. log10 1 = 0

D. log (1 + 2 + 3) = log 1 + log 2 + log 3


E. All are correct

41 | P a g e For classes | Shortcut workshops | mocks | books Cetking –


09594441448 | 09930028086| 09820377380 | www.cetking.com
MBA CET 2024 Expected paper
42. A man's speed with the current is 15 km/hr and the speed of the current is 2.5 km/hr. The man's speed
against the current is:
A. 8.5 km/hr
B. 9 km/hr
C. 10 km/hr
D. 12.5 km/hr
E. 15 km/hr

43. A boat covers a certain distance downstream in 1 hour, while it comes back in 1 hours. If the speed of
the stream be 3 kmph, what is the speed of the boat in still water?
A. 12 kmph
B. 13 kmph
C. 14 kmph
D. 15 kmph
E. None of these

44. If a quarter kg of potato costs 60 paise, how many paise will 200 gm cost?
A. 48 paise
B. 54 paise
C. 56 paise
D. 72 paise
E. 40 paise.

45. A wheel that has 6 cogs is meshed with a larger wheel of 14 cogs. When the smaller wheel has made
21 revolutions, then the number of revolutions mad by the larger wheel is:
A. 4
B. 9
C. 12
D. 49
E. 20

46. Find the value of x?

A. 2
B. 4
C. 3
D. 8
E. 5

47. Find the value of x?


If 3 (x - y) = 27 and 3(x + y) = 243
A. 2
B. 4
C. 3
D. 8
E. 5

48. Let N be the greatest number that will divide 1305, 4665 and 6905, leaving the same remainder in each
case. Then sum of the digits in N is:
A. 4
B. 5
C. 6
42 | P a g e For classes | Shortcut workshops | mocks | books Cetking –
09594441448 | 09930028086| 09820377380 | www.cetking.com
MBA CET 2024 Expected paper
D. 8
E. 7

49. The captain of a cricket team of 11 members is 26 years old and the wicket keeper is 3 years older. If
the ages of these two are excluded, the average age of the remaining players is one year less than the average
age of the whole team. What is the average age of the team?
A. 23 years
B. 24 years
C. 25 years
D. 26 years
E. 27 years

50. It was Sunday on Jan 1, 2006. What was the day of the week Jan 1, 2010?
A. Sunday
B. Saturday
C. Friday
D. Wednesday
E. None of these

43 | P a g e For classes | Shortcut workshops | mocks | books Cetking –


09594441448 | 09930028086| 09820377380 | www.cetking.com
MBA CET 2024 Expected paper
1. Correct Option: D
In the given statements, statement II states India’s strength as a major agricultural nation with a large poultry
industry and statement II discusses what India has done to deal with avian influenza. Applying the elimination
technique, let’s check the relevance of each of the options one by one.
Possibility 1. Although India has implemented an action plan formulated by the Centre’s Department of
Animal Husbandry, Dairying and Fisheries to deal with avian influenza, it is a major agricultural nation with a
large poultry industry.
Or Although India is a major agricultural nation with a large poultry industry, it has implemented an action
plan formulated by the Centre’s Department of Animal Husbandry, Dairying and Fisheries to deal with avian
influenza. We can observe that neither of the above two formations expresses a meaningful context. This
eliminates option A.
Possibility 2. As a major agricultural nation with a large poultry industry, India has implemented an action
plan formulated by the Centre’s Department of Animal Husbandry, Dairying and Fisheries to deal with avian
influenza. Clearly, the above synthesis is both grammatically and contextually correct and keeps the flow of
the sentence intact.
Possibility 3. Being a major agricultural nation with a large poultry industry, India has implemented an action
plan formulated by the Centre’s Department of Animal Husbandry, Dairying and Fisheries to deal with avian
influenza. Evidently, the above synthesis is also both grammatically and contextually correct and coherent.
Option D is hence the correct answer.

2. Correct Option: B
The given statements seem to be effects of a common cause. Applying the elimination technique, let’s check
the relevance of each of the options one by one.
Possibility 1. On account of technological change that banks in India have embraced, the onus is on them to
integrate inter-generational legacy systems across branches, ATMs and online banking networks into one
seamless and secure whole. We can observe here that though the formation above is grammatically correct, it
doesn’t express a meaningful context. This hence eliminates option C.
Possibility 2. Besides the onus on banks to integrate integrate inter-generational legacy systems across
branches, ATMs and online banking networks into one seamless and secure whole, they in India have
embraced technological change. Once again we have here a construction that is grammatically correct but
logically flawed. This hence eliminates option A and D as well.
Possibility 3. With banks in India having embraced technological change, the onus is on them to integrate
inter-generational legacy systems across branches, ATMs and online banking networks into one seamless and
secure whole. Clearly, the synthesis made above is both grammatically and contextually correct and keeps the
flow of the sentence intact.

3. Correct Option: E
In the above statements, while statement II affirms an occurrence of an event, statement I re-affirms it in a bit
elaborative manner. Applying the elimination technique, let’s check the relevance of each of the options one
by one.
Possibility 1: As long as the long queues in the outpatient clinics at medical colleges and general hospitals are
often the first indicator of the beginning of the epidemic season in Kerala, usually, dengue cases begin to peak
with the arrival of the southwest monsoon. Clearly, the above synthesis is contextually flawed and obstructs
the flow of the sentence. The synthesis is hence invalid and thus eliminates option C.
Possibility 2: Though, usually, dengue cases begin to peak with the arrival of the southwest monsoon, the long
queues in the outpatient clinics at medical colleges and general hospitals are often the first indicator of the
beginning of the epidemic season in Kerala. Usage of though in a clause is generally followed by a
contradiction whereas the synthesis above doesn’t indicate a contradiction at all. Clearly, the phrase/connector
‘b’ also stays invalid. Options B and D also get eliminated.
Possibility 3: Surpassing the beginning of the epidemic season in Kerala,the long queues in the outpatient
clinics at medical colleges and general hospitals are often the first indicator, usually, dengue cases begin to
peak with the arrival of the southwest monsoon. As we can observe that the synthesis above doesn’t make a

44 | P a g e For classes | Shortcut workshops | mocks | books Cetking –


09594441448 | 09930028086| 09820377380 | www.cetking.com
MBA CET 2024 Expected paper
meaningful sentence either, we can conclude that none of the constructions stays logical here. Option E is
hence the correct answer.

4. Correct Option: A
Both the statements talk about the effects of the same thing i.e. higher interest rates. We need connectors that
can portray this relationship.
Option B: Even though is incorrect.
Option C: Least does not fit in.
Option D: Otherwise does not fit in.
Only option A fits in well:
I. Not only do higher interest rates make equity investments unattractive as it weighs on earnings growth, they
lead to tightening of earnings yield spread over bond yields.
II. Higher interest rates make equity investments unattractive as it weighs on earnings growth besides
tightening of earnings yield spread over bond yields.
Thus, option A is correct.

5. Correct Option: C
Statement A talks about a situation while statement B talks about dealing with it in an appropriate manner.
Option A: Too does not fit in.
Option B: Neither fits in.
Option D: Neither fits in.
Only Option C fits in:
I. Instead of confiscating all properties belonging to a fugitive economic offender, a prudent step would be to
quantify the outstanding loan amount and accordingly, only those properties must be confiscated that have
been acquired through proceeds of crime.
II. Instead of confiscating all properties belonging to a fugitive economic offender, a prudent step would be to
quantify the outstanding loan amount and hence, only those properties must be confiscated that have been
acquired through proceeds of crime.
Hence, option C is correct.

Solution In the above example you will observe


that the flow of logic is in the form of a time
sequence which flows from the oldest time
period to a more contemporary time period.
Therefore, Sentence C will be the first sentence.
Sentence B expands upon the ―simple model‖
proposed, hence, it will be the sentence
following
C. The next sentence in the order of chronology
is C— nearly a century passed, while the last
sentence will be A which completes the sequence from older time to contemporary time thus giving us the
answer as CBDA.
Correct order is CBDA.
6. Answer is C
7. Answer is B
8. Answer is D
9. Answer is A

10. Read sentences C and D carefully. Sentence D


contains the noun phrase ―a salmonella
bacterium‖ and Sentence C contains the noun
phrase ―salmonella infections‖. What is the

45 | P a g e For classes | Shortcut workshops | mocks | books Cetking –


09594441448 | 09930028086| 09820377380 | www.cetking.com
MBA CET 2024 Expected paper
relationship between the two? Since the phrase
―a salmonella bacterium‖ introduces the
bacterium, it should logically precede the
phrase ―salmonella infections‖. Therefore, the
sentence that contains the phrase ―a salmonella
bacterium‖ should come before the sentence
that contains the phrase ―salmonella
infections‖. So, Sentence D should precede
Sentence C! Once you have a link between two
sentences, look at the answer choices to see if
you are on the right track. If you are, then you
have the right answer [(Option (d) BDCA)] and
it is time to move on to the next exercise.

11. Correct Option: A. ‘Mixed bag of’ is an idiomatic expression that refers to something that has both
positive and negative qualities or aspects. Ex. The session after lunch was a mixed bag of some good and bad
performances. Out of the choices, the statement given in option A confirms the usage of the expression
‘Mixed bag of’ rightfully in the context. Option A is hence the correct answer.
12. Correct Option: E. According to the passage all statements are incorrect.
13. Correct Option: D. References. Last paragraph, 5th and 6th sentences. The budget also initiated the
decentralisation of funding in GRB, thus shifting the onus for budgeting and implementation from the Central
Ministry to State counterparts. While this did empower the States to come up with women-specific policies as
per their respective challenges, the obvious downside was the risk that States could choose to not prioritise
gender in their budgeting. The author does not evidently support the idea of decentralisation of funding in
GRB by presenting a disadvantageous aspect of the move.
14. Correct Option: E. Exacerbate (Verb): To make (a problem, bad situation, or negative feeling) worse.
15. Correct Option: C. Onus (Noun): Onus, as a formal word is used for responsibility or obligation.
16. Correct Option: B. Reference 1st paragraph, 1st sentence During the 2014 national and State elections,
for the first time women’s safety and empowerment were topics of debate, marking a significant shift in how
gender concerns are viewed by the political class as well as by voters in India.
17. Correct Option: A. Reference. 1st paragraph, 3rd sentence. The direct link between empowering women
and alleviating poverty, increasing productivity, and combating climate change is well-recognised.

18. Correct Option: A. Refer to, “The Budget sounded the bugle for fostering rural entrepreneurship for
those depending on agriculture and traditional industries by announcing the setting up of 80 Livelihood
Business Incubators and 20 Technology Business Incubators this fiscal with the aim to develop 75,000 skilled
entrepreneurs in agro-rural industry sector.” It is clear that the government now wants the startup founders to
go beyond the metro cities in order to make sure that the agriculture sector also gets the desired focus from
such people. It will make it a point that the rural economy will also develop in the country. Among the given
options, Option A explains this fact and that is why it is our pick here whereas the rest can be eliminated for
the fact that they do not follow from the information given in the passage. This makes Option A the correct
choice among the given options.
19. Correct Option: D. It has been described in the passage that the government has announced a number of
steps in order to boost the startup economy of the country. The government has announced tax exemptions and
also more networking in order to help the startups develop more and more. Among the given options, we can
easily pick up Option D whereas Option A is correct in the first part but for the second part, we can say that no
question has been put forward by the government for the startup sector of the country. Other options can be
eliminated since they do not at all follow from the passage. This makes Option D the correct choice among the
given options.
20. Correct Option: D. Statement I is not correct for the fact that the new channel has been proposed to se
set up and run by the startup companies themselves so that they can utilize the platform properly. Refer to, “A
TV channel exclusively for start-ups under the Doordarshan boutique will be designed and executed by start-

46 | P a g e For classes | Shortcut workshops | mocks | books Cetking –


09594441448 | 09930028086| 09820377380 | www.cetking.com
MBA CET 2024 Expected paper
ups themselves and will also serve as a platform for promoting them, discussing issues affecting their growth,
match making with VCs and for funding and tax planning, came as a pleasant surprise to many.”. Statement II
is also not correct because it is said that the new channel will be financed by the public sector banks whereas
there is no such reference in the passage to that effect. It can be eliminated. Statement III is correct because
from the above quoted lines it can be understood that the new channel will try to match the startup founders
with the VCs so that they can raise capital and also plan the taxes so that there is tax saving by these small
companies. This makes Option D the correct choice among the given options.
21. Correct Option: C. We can see from the passage that the author is not overjoyed with the fact that the
government has announced a new channel that will be dedicated to the startup sector of the country. He wants
to see if the government is actually able to execute this idea at the end as it will be difficult to bring everything
in one place to get this going. Among the given options, Option C explains this reaction of the author whereas
the rest can be eliminated because they do not follow from the passage. This makes Option C the correct
choice among the given options.
22. Correct Option: B. Statement I is correct for the fact that Angel Tax is a kind of tax that is levied on the
startups that raise capital from the angel investors. So, it is correct that the companies that raise capital from
this source only will be subjected to this particular tax. Statement II is not correct because it is said that
Section 56 of the Income Tax Act is regarding the Angel Tax that is levied on companies raising capital from
angel investors. So, it cannot be said that there is no such provision in the Income Tax Act of the country.
Statement III is not correct for the fact that the Angel Tax is not going to be levied on any startup from now
onwards as has been announced in the recent Budget of the government. So, it is applicable for each and every
company and not for them who have decided to shut down the business. This makes Option B the correct
choice among the given options.

23. Answer B. verb: institutionalise. establish (something, typically a practice or activity) as a convention or
norm in an organization or culture.
24. Answer A. A bugle is a simple brass musical instrument that looks like a small trumpet. Bugles are often
used in the army to announce when activities such as meals are about to begin.
25. Answer E. Definition of contentious: likely to cause disagreement or argument

47 | P a g e For classes | Shortcut workshops | mocks | books Cetking –


09594441448 | 09930028086| 09820377380 | www.cetking.com
MBA CET 2024 Expected paper

1. e. RBI is facing down


2. c. person sitting next and facing down so kavit is next to Ronit facing down
3. b. Two people
4. e. Sumit
5. b. Two

Rank 1 2 3 4 5 6
Student I N A D J L
Place Madurai chennai nag trichy tiru Coimb
Dress (Rs) 400 350 200 450 300 250

6. c. 300
7. b. Chennai
8. d. 3rd Rank
9. d. four
10. a. Lakshit

Rack 8 7 6 5 4 3 2 1
Box Box P Box T Box Q Box U Box W Box R Box V Box S
Balls 44 67 36 81 93 72 59 89

11. 2. Two boxes away


12. 1. P is 44 balls
13. 1. P is 44 balls
14. 3. R is in 3rd Rack
15. 2. Q has 36 balls

Name Gender Place Car


D Girl Del X
G Boy Del X
A Girl Ag Z
H Boy Ag Z
F Girl Ch Y
C Ch Y
E Del X
B Boy

16. Option 3. Gender can be found for all except C and E.


17. Option 5. Cannot be determined. B can be in X or Y.
18. Option 5. Cannot be determined. Gender of C is not clear.
19. Option 5. All combination are wrong
20. Option 5. Cannot be determined. Gender of E is not clear.

21. 5. None of these


22. 4. W Y
23. 5. None of these
24. 2. T is brother
25. 1. Y sits opposite

48 | P a g e For classes | Shortcut workshops | mocks | books Cetking –


09594441448 | 09930028086| 09820377380 | www.cetking.com
MBA CET 2024 Expected paper
Po Cy Fg Cv In Pu
Mc Fr So Lo Sw Acc

26. 4. Accord
27. 2. Polo
28. 2. Polo
29. 4. Figo
30. 4. Santro

49 | P a g e For classes | Shortcut workshops | mocks | books Cetking –


09594441448 | 09930028086| 09820377380 | www.cetking.com
MBA CET 2024 Expected paper
1. the correct answer is "Option 3".
The logic followed here is:
(1st number - 2nd number)2 × 2 = 3rd number
For (24, 10, 392):
(24 - 10)2 × 2 = 14² × 2 = 196 × 2 = 392
Option 1: (26, 12, 369)
(26 - 12)2 × 2 = 142 × 2 = 196 × 2 = 392 ≠ 369
Option 2: (27, 15, 480)
(27 - 15)2 × 2 = 122 × 2 = 144 × 2 = 288 ≠ 480
Option 3: (29, 18, 242)
(29 - 18)2 × 2 =112 × 2 = 121 × 2 = 242
Option 4: (21, 18, 234)
(21 - 18)2 × 2 = 32 × 2 = 9 × 2 = 18 ≠ 234
Hence, the correct answer is "Option 3".

2. the correct answer is "Option 1".


1 2 3 4 5 6 7 8 9 10 11 12 13
A B C D E F G H I J K L M
Z Y X W V U T S R Q P O N
26 25 24 23 22 21 20 19 18 17 16 15 14

By checking options,
(1) NPSV

(2) QTWZ

(3) XADG

(4) BEHK

Hence, NPSV is the odd one.

3. the correct answer is "Option 1".

Given: BOARD, BOARW, BOAIW, ?, BLZIW, YLZIW


The logic followed here is:
• one letter starting from the last letter of the word BOARD is changed to its opposite letter in each step as shown below,

Hence, BOZIW is the correct answer.

50 | P a g e For classes | Shortcut workshops | mocks | books Cetking –


09594441448 | 09930028086| 09820377380 | www.cetking.com
MBA CET 2024 Expected paper

4. the correct answer is "Option 3".

Given: ‘AND’ is written as ‘C-LP-F’ and ‘NOR’ is coded as ‘P-MQ-T’.


The logic followed here is:

And,

Similarly,

Hence, D-SW-V is the correct answer.

5. the correct answer is "Option 4".


Awful : Nice : : Nomadic : Settled

1. Success 2. Surreal 3. Succumb 4. Suction 5. Surrogate 6. Surprise


6. Option 4 is the correct order: 1, 3, 4, 6, 2, 5

7. the correct answer is "Option 5".

Given: 'SAFETY' is coded as '80' and 'EXPAND' is coded as '68'.


The logic followed here is:
• Sum of positional values + no. of consonants in the word
Letters S A F E T Y

Positional Values 19 1 6 5 20 25

Sum of Positional Values 19 + 1 + 6 + 5 + 20 + 25 = 76


Number of consonants = 4
Now, 76 + 4 = 80
Letters E X P A N D

Positional Values 5 24 16 1 14 4

Sum of Positional Values 5 + 24 + 16 + 1 + 14 + 4 = 64


Number of consonants = 4
Now, 64 + 4 = 68
Similarly,
Letters G A T H E R

Positional Values 7 1 20 8 5 18

Sum of Positional Values 7 + 1 + 20 + 8 + 5 + 18 = 59


Number of consonants = 4
Now, 59 + 4 = 63
Hence, 63 is the correct answer.

51 | P a g e For classes | Shortcut workshops | mocks | books Cetking –


09594441448 | 09930028086| 09820377380 | www.cetking.com
MBA CET 2024 Expected paper
Given: BOARD, BOARW, BOAIW, ?, BLZIW, YLZIW
The logic followed here is:
• one letter starting from the last letter of the word BOARD is changed to its opposite letter in each step as shown below,

Hence, BOZIW is the correct answer.

8. Option 4 is correct
Atleast two means two or there. So all middle values 22 + 15 + 7 + 5 = 49

9. Correct is a. sh na pn.

10. Option 4 is correct


She (girl) is my (Raghav) father's mother's daughter's only brother's daughter, implies Raghav's father is the father of that girl.
Therefore, the final family tree is as follows,

Hence, Raghav's father is the husband of the mother of girl.

11. Correct option is 3. Son in law

12. Correct option is 4.

52 | P a g e For classes | Shortcut workshops | mocks | books Cetking –


09594441448 | 09930028086| 09820377380 | www.cetking.com
MBA CET 2024 Expected paper
According to the question, X × (X + 1) = 1260 X2 + X - 1260 = 0 By solving the above equation we get, X = 35 and - 36 ⇒ X = 35
(as X is a positive number) And, X + Y = 106 ⇒ Y = 106 - X Y = 106 - 35 Y = 71 Hence, 71 is the correct answer.

13. Option 3 : b, f, b, e, c, k, e
Given: b _ c e _ k b _ c _ f k b b _ e f _ b b c _ f k
By checking options, (1) b, f, b, e, c, e, k → b b c e f k - b b c e f k - b b c e f e - b b c k f k (2) b, e, b, f, c, k, e → b b c e e k - b b c f
f k - b b c e f k - b b c e f k (3) b, f, b, e, c, k, e → b b c e f k - b b c e f k - b b c e f k - b b c e f k Here, 'bbcefk' is repeated (4) b, f, b,
c, e, k, e → b b c e f k - b b c c f k - b b e e f k - b b c e f k Only options (3) b, f, b, e, c, k, e forms pattern when substituted. Hence,
b, f, b, e, c, k, e is the correct answer.

14. Option E. (iv) condition is not specified. So move to (b). But this is also not fulfilled so he will not be selected for any job

15. Option 4 is correct :


Given: 21 ∶ 445 ∶∶ 24 ∶ ? The logic followed here is: 21 : 445 → 21 × 21 + 4 = 441 + 4 = 445 Similarly, 24 : ? → 24 × 24 + 4 = 576 +
4 = 580 Hence, 580 is the correct answer.

Read the given statements and conclusions carefully. Assuming that the information given in the statements is true, even if it appears
to be at variance with commonly known facts, decide which of the given conclusions logically follow(s) from the statements.

16. Option 5. All conclusions follow.

17. Option 2 is odd man out


The pattern followed here is: The difference between the numbers in each pair is 1234. Let's check each option:
5237 : 4003 → 5237 - 4003 = 1234
4327 : 2003 → 4327 - 2003 = 2324
3527 : 2293 → 3527 - 2293 = 1234
2347 : 1113 → 2347 - 1113 = 1234
Hence, 4327 : 2003 is the odd one.

18. Correct Option: B. Doctors and lawyers are entirely different. But, both are Professionals.
19. Correct Option: D. Hydrogen is a constituent of both Water and Atmosphere. Water is present in Atmosphere.
20. Correct Option: C. Dictionary is also a type of Book . But printer has a separate identity

21. Correct Option: B Venn Diagram Method:

22. Correct answer B


23. Correct answer D
24. Correct answer E
25. Correct answer C
26. Correct answer D

53 | P a g e For classes | Shortcut workshops | mocks | books Cetking –


09594441448 | 09930028086| 09820377380 | www.cetking.com
MBA CET 2024 Expected paper

27. Correct Option: A Venn Diagram Method:

28. Correct Option: D Venn Diagram Method:

29. Correct Option: B Venn Diagram Method:

30. Correct answer A


31. Correct answer A
32. Correct answer C
33. Correct answer B

34. Correct Option: B 6219 means ‘Sachin is a cricketer’

And,
2646 means ‘He played from Mumbai’

54 | P a g e For classes | Shortcut workshops | mocks | books Cetking –


09594441448 | 09930028086| 09820377380 | www.cetking.com
MBA CET 2024 Expected paper
Similarly,
‘Mumbai is very famous’ means ?

Hence, the correct answer is "6246".

35. Correct answer 3


Dentist : Hospital is same as Professor : University

The following pie-charts show the percentage distribution of the total employees of two Companies A and B
in different departments, and the table shows the ratio of Male to Female employees in all the departments of

Company A and B. The total number of employees working in Company A and B are 8000 and 7500
respectively.
6. What is the total number of Female
employees in D5 of Company A and B
together?
(a) 705 (b) 710 (c) 715 (d) 720 (e) 725

7. The total number of Female employees in D1 of Company B is approximately how much per cent more
than the number of Female employees in D1 of Company A?
(a) 5% (b) 7.5% (c) 15% (d) 22.5% (e) 30%

55 | P a g e For classes | Shortcut workshops | mocks | books Cetking –


09594441448 | 09930028086| 09820377380 | www.cetking.com
MBA CET 2024 Expected paper

8. What is the difference between the total Male employees of Company A and the total Female employees of Company
B?
(a) 1230 (b) 1232 (c) 1234 (d) 1236 (e) 1238

56 | P a g e For classes | Shortcut workshops | mocks | books Cetking –


09594441448 | 09930028086| 09820377380 | www.cetking.com
MBA CET 2024 Expected paper
9. The average number of Male employees in D1 and D2 of Company B is approximately what percentage of the
average number of Female employees in D5 and D6 of Company A?
(a) 177.5% (b) 197.5% (c) 212.5% (d) 217.5% (e) 227.5%

10. The total number of Females working in Company A is approximately what percentage of total employees of
Company A?
(a) 42.12% (b) 43.48% (c) 44.24% (d) 45.64% (e) 46.86%

57 | P a g e For classes | Shortcut workshops | mocks | books Cetking –


09594441448 | 09930028086| 09820377380 | www.cetking.com
MBA CET 2024 Expected paper

Study the following information carefully and answer the questions given below:

In an institute there are 5600 students, the ratio of the number of girls to the number of boys is 3 : 4 respectively. All the
students are enrolled in different programmes viz. SSC, Banking, CLAT and F–CAT. 20% of the total number of boys are
enrolled in CLAT programme. The number of girls enrolled in Banking is five–fourths of the number of boys taking the same.

One fourth of the total number of girls enrolled in F–CAT programme. The total number of students enrolled in F– CAT
rogramme is 1400. Two–fifths of the boys enrolled in SSC programme and the remaining boys are enrolled in Banking
programme. The girls enrolled in SSC are three fourth of the number of boys enrolled in the same. The remaining girls are
enrolled in CLAT programme.

6. What is the respective ratio of the number of boys enrolled in CLAT to the number of boys enrolled in SSC? 1) 1 : 2 2)
1 : 3 3) 2 : 3 4) 4 : 5 5) 3 : 4

7. What is the number of girls enrolled in CLAT? 1) 210 2) 260 3) 280 4) 180 5) 240

8. What is the total number of students enrolled in Banking?


1) 1060 2) None of these 3) 1080 4) 1040 5) 1020

9. The number of girls enrolled in F–CAT is approximately what percent of the total number of students enrolled in SSC?
1) 24.8% 2) 26.8% 3) 36.8% 4) 32.9% 5) 29.4%

10. The number of boys enrolled in CLAT and SSC together is approximately what percent more or less than the number
of girls enrolled in SSC?
1) 100% more 2) 100% less 3) 125% more
4) Other than given options 5) 75% less

58 | P a g e For classes | Shortcut workshops | mocks | books Cetking –


09594441448 | 09930028086| 09820377380 | www.cetking.com
MBA CET 2024 Expected paper

6. 1

Required ratio = 640 : 1280 = 1 : 2

7. 5

Required answer = 240 8. 3


Required answer = 600 + 480 = 1080

9. 2

Required percentage = 600/ (1280 + 960) * 100 = 600/2240


* 100 = 26.8%

10. 1
Required percentage more = {(1280 + 640) - 960} / 960 *
100 = 100% more

Directions Q1 to Q5- there are five travelling companies. And the following table gives the percentage
distribution of all customers according to their mode of transportation

Q1. The total number of passengertravels through kali travels is 300, and the passenger travels by bus and air for royal
cruiser is equal to the passenger travel by all the modes other than Air from kali travels. Find the ratio of traveler by Train
from Royal cruiser to Bus traveler of kali travels.
(a)138/225 (b)128/225 (c)124/123 (d)152/152 (e)None

59 | P a g e For classes | Shortcut workshops | mocks | books Cetking –


09594441448 | 09930028086| 09820377380 | www.cetking.com
MBA CET 2024 Expected paper
Q2. If the total number of passenger is 500 from the royal
cruise and same for Iana travels. Find the difference of
people travelling by Bus from these two travel agencies.
(a)44 (b)54 (c)45 (d)36 (e)None of the above

Q13. From the Kingfisher travels passenger travelling by train and ship is equal to the passenger travelling by car. Then how
much percentage of passenger is travelling by Bus?
(a)15 (b)16 (c)17 (d)18 (e) None of the above

Q14. Kingfisher passengers are 60% more than the Royal cruiser total passenger. Then the passenger travelling by bus from
kingfisher travels is how much percent more/less than that of royal travel travelling through same mode of transport.
(a)35 (b)37.37 (c)36.84 (d)47 (e)48.26

Q15. How many passengers are travelling by Nainital travels if car and bus traveler of Nanital travels is equal to the Air traveler
of Kingfisher travels.
(a)500
(b) 550
(c) 560
(d) 450
(e) None of the above

60 | P a g e For classes | Shortcut workshops | mocks | books Cetking –


09594441448 | 09930028086| 09820377380 | www.cetking.com
MBA CET 2024 Expected paper

61 | P a g e For classes | Shortcut workshops | mocks | books Cetking –


09594441448 | 09930028086| 09820377380 | www.cetking.com
MBA CET 2024 Expected paper
16. To find out the share of Y out of Rs 1820, which of the following statements is/are
sufficient/necessary?
A. The share of X is 1.8 times the combined share of Y and Z.
B. The share of Y is3/11 of the combined share of X and Z.
C. The share of Z is 1/6of the combined share of X and Y.
(a) Statements A and B together are sufficient
(b) Statement A and C together are sufficient
(c) Statements B and C together are sufficient
(d) Either statement B alone or statements A and C together are sufficient
(e) None of these

17. A sum of money Rs 2550 is to be distributed among Knahiya, Varun and Rajneesh. What
will be the share of Varun?
A. Knhiya’s share is 1.5 times Varun’s share.
B. Rajneesh’s share is half the share of Knahiya and Varun together.
C. The share of Knahiya is Rs 340 more than Varun.
(a) Either A and B or A and C together are sufficient
(b) Only A and B (c) Only A and C (d) All are required
(e) Even all together are not sufficient

18. To find the temperature on Monday which of the following information is sufficient?
A. The average temperature for Monday, Tuesday and Wednesday was 38° C.
B. The average temperature for Tuesday, Wednesday and Thursday was 43° C.
C. The temperature on Tuesday and Thursday was 45° C each.
(a) Only A and B are sufficient (b) Only B and C
(c) A and either B or C (d) C and either A or B
(e) All the three together are sufficient

62 | P a g e For classes | Shortcut workshops | mocks | books Cetking –


09594441448 | 09930028086| 09820377380 | www.cetking.com
MBA CET 2024 Expected paper
19 20.

21. Series Pattern Given Series


23 23
23 + (12 + 1) = 25 25
25 + (22 + 1) = 30 30
30 + (32 + 1) = 40 40
40 + (42 + 1) = 57 57
57 + (52 + 1) = 83 83

22. Series Pattern Given Series


13 13
13 × 2 – 2 = 24 24
24 × 4 – 4 = 92 92
92 × 6 – 6 = 546 546
546 × 8 – 8 = 4360 4360
4360 × 10 – 10 = 43590 43590

23 Correct Option: B
By the applying BODMAS rule, we get
Given expression = (2700 – 240) / (1120 + 110) ⇒ (2460 / 1230) ⇒ 2

24. Correct Option: C


Let (3/8) of 168 × 15 ÷ 5 + A = 549 ÷ 9 + 235
Then, 63 × 15 ÷ 5 + A = 549 ÷ 9 + 235
⇒ 63 × 3 + A = 61 + 235
⇒ 189 + A = 296 , A = 107.

25

63 | P a g e For classes | Shortcut workshops | mocks | books Cetking –


09594441448 | 09930028086| 09820377380 | www.cetking.com
MBA CET 2024 Expected paper

26.

27.

64 | P a g e For classes | Shortcut workshops | mocks | books Cetking –


09594441448 | 09930028086| 09820377380 | www.cetking.com
MBA CET 2024 Expected paper

65 | P a g e For classes | Shortcut workshops | mocks | books Cetking –


09594441448 | 09930028086| 09820377380 | www.cetking.com
MBA CET 2024 Expected paper
Answers
1. Answer: Option E Evacuating low-lying areas and
rushing in relief to the affected areas clearly indicates that
floods have occurred in the area.
2. Answer: Option B The increase in number of asthma
cases must have alerted the authorities to take action to
control air pollution that triggers the disease.
3. Answer E. All 3 follows, even though III is long
term solution but the problem is also long term.
4. (2) (A) is irrelevant and (C) supports the given
argument. (B) weakens the argument by saying that
since more people consume liquor in rural areas hence
it is normal that the cases of food poisoning are more
in rural areas. Similarly (D) also weakens the
argument.
5. (3) Only (C) supports the argument saying that there
are many unauthorized spurious liquor shops in the rural
areas due to which the low quality liquor is supplied in
rural areas.
6. . (3) (1), (2) and (4) are irrelevant as nothing is
mentioned about them. The district administration has
issued a circular to make the farmers aware of hazards
that indiscriminate use of pesticides poses to ground
water and plead them to refrain from the same. Hence
(3) is an assumption in the given statement.

7. (1) Only I argument is correct. I. Yes, those who


violate the law, must be punished.
8. (4) If (4) is not true we cannot reach the
conclusion stated above.
9. (1) Only (1) is implicit in the given statement as it is
a desired effect. (2) and (3) are not relevant as they are
neither the root cause nor the desired effect.
10. (1) Only I assumption is correct.. The student may stop using cell phone in the college premises

66 | P a g e For classes | Shortcut workshops | mocks | books Cetking –


09594441448 | 09930028086| 09820377380 | www.cetking.com
MBA CET 2024 Expected paper
1. Answer: Option D
Let the son's present age be x years. Then, man's present age = (x + 24) years.
(x + 24) + 2 = 2(x + 2)
x + 26 = 2x + 4
x = 22.

2. Answer: Option B
Angle traced by hour hand in 17/2 hrs = 360/12 x 17/2 = 255°.
Angle traced by min. hand in 30 min. = 360/60 x 30° = 180°.
Required angle = (255 - 180)° = 75°.

3. Answer: Option B
For managing, A received = 5% of Rs. 7400 = Rs. 370.
Balance = Rs. (7400 - 370) = Rs. 7030.
Ratio of their investments = (6500 x 6) : (8400 x 5) : (10000 x 3)
= 39000 : 42000 : 30000
= 13 : 14 : 10
B's share = Rs. 7030 x 14/37 = Rs. 2660.

4. Answer: Option A

5. Answer: Option A

6. Answer: Option D
Let the speed of two trains be 7x and 8x km/hr.
Then, 8x = 400/4 = 100
x= 100 / 8= 12.5
Speed of first train = (7 x 12.5) km/hr = 87.5 km/hr.
7. Answer: Option C
67 | P a g e For classes | Shortcut workshops | mocks | books Cetking –
09594441448 | 09930028086| 09820377380 | www.cetking.com
MBA CET 2024 Expected paper
The word 'LEADING' has 7 different letters.
When the vowels EAI are always together, they can be supposed to form one letter.
Then, we have to arrange the letters LNDG (EAI).
Now, 5 (4 + 1 = 5) letters can be arranged in 5! = 120 ways.
The vowels (EAI) can be arranged among themselves in 3! = 6 ways.
Required number of ways = (120 x 6) = 720.

8. Answer: Option D
We have: (l - b) = 23 and 2(l + b) = 206 or (l + b) = 103.
Solving the two equations, we get: l = 63 and b = 40.
Area = (l x b) = (63 x 40) m2 = 2520 m2.

9. Answer: Option B
Let the ten's digit be x and unit's digit be y.
Then, (10x + y) - (10y + x) = 36
9(x - y) = 36
x - y = 4.

10. Answer: Option C


Let the shares of A, B, C and D be Rs. 5x, Rs. 2x, Rs. 4x and Rs. 3x respectively.
Then, 4x - 3x = 1000
x = 1000.
B's share = Rs. 2x = Rs. (2 x 1000) = Rs. 2000.

11. Answer: Option C


Suppose, first pipe alone takes x hours to fill the tank .
Then, second and third pipes will take (x -5) and (x - 9) hours respectively to fill the tank.
1/x + 1/(x - 5) = 1/(x - 9)
x - 5 + x / x(x - 5) = 1/(x - 9)
(2x - 5)(x - 9) = x(x - 5)
x2 - 18x + 45 = 0
(x - 15)(x - 3) = 0
x = 15. [neglecting x = 3]

12. Answer: Option B

13. Answer: Option C


A : B = 60 : 45.
68 | P a g e For classes | Shortcut workshops | mocks | books Cetking –
09594441448 | 09930028086| 09820377380 | www.cetking.com
MBA CET 2024 Expected paper
A : C = 60 : 40.
B/C = B/A x A/c = 45 / 60 x 60/40 = 45 / 40 = 90/80
B can give C 10 points in a game of 90.

14. Answer: Option C


In two throws of a dice, n(S) = (6 x 6) = 36.
Let E = event of getting a sum ={(3, 6), (4, 5), (5, 4), (6, 3)}.
P(E) =4/36 = 1/9

15. Answer: Option C

16. Answer: Option B


Explanation:
(a) Since loga a = 1, so log10 10 = 1.
(b) log (2 + 3) = log 5 and log (2 x 3) = log 6 = log 2 + log 3
log (2 + 3) log (2 x 3)
(c) Since loga 1 = 0, so log10 1 = 0.
(d) log (1 + 2 + 3) = log 6 = log (1 x 2 x 3) = log 1 + log 2 + log 3.
So, (b) is incorrect.

17. Answer: Option C


Man's rate in still water = (15 - 2.5) km/hr = 12.5 km/hr.
Man's rate against the current = (12.5 - 2.5) km/hr = 10 km/hr.

18. Answer: Option D


Let the speed of the boat in still water be x kmph. Then,
Speed downstream = (x + 3) kmph,
Speed upstream = (x - 3) kmph.
(x + 3) x 1 = (x - 3) x 3/2
2x + 6 = 3x - 9
x = 15 kmph.

19. Answer: Option A


Let the required weight be x kg.
Less weight, Less cost (Direct Proportion)
250 : 200 :: 60 : x => 250 x x = (200 x 60)
x= (200 x 60)/250
69 | P a g e For classes | Shortcut workshops | mocks | books Cetking –
09594441448 | 09930028086| 09820377380 | www.cetking.com
MBA CET 2024 Expected paper
x = 48.

20. Answer: Option B


Let the required number of revolutions made by larger wheel be x.
Then, More cogs, Less revolutions (Indirect Proportion)
14 : 6 :: 21 : x 14 x x = 6 x 21
x = 6 x 21
14
x = 9.

21. Answer: Option A

22. Answer: Option C


3x - y = 27 = 33 x - y = 3 ....(i)
3x + y = 243 = 35 x + y = 5 ....(ii)
On solving (i) and (ii), we get x = 4.

23. Answer: Option A


N = H.C.F. of (4665 - 1305), (6905 - 4665) and (6905 - 1305)
= H.C.F. of 3360, 2240 and 5600 = 1120.
Sum of digits in N = ( 1 + 1 + 2 + 0 ) = 4

24. Answer: Option A


Let the average age of the whole team by x years.
11x - (26 + 29) = 9(x -1)
11x - 9x = 46
2x = 46
x = 23.
So, average age of the team is 23 years.

25. Answer: Option C


On 31st December, 2005 it was Saturday.
Number of odd days from the year 2006 to the year 2009 = (1 + 1 + 2 + 1) = 5 days.
On 31st December 2009, it was Thursday.
Thus, on 1st Jan, 2010 it is Friday.

11.

70 | P a g e For classes | Shortcut workshops | mocks | books Cetking –


09594441448 | 09930028086| 09820377380 | www.cetking.com
MBA CET 2024 Expected paper
1. Answer 3: Except 3, all other figure have 3 unshaded and one shaded leaves.
2. Answer 4: In all 4 elements direction of sector is same as arrow
3. Answer 3: Direction of arrow in figure 2 and 3 are same.
4. Answer 5: From figures (i) and (ii) we conclude that 2, 6 and 4 dots appear adjacent to 3 dots. Hence, either 1 or 5 dots
may appear opposite 3 dots. Thus, if the dice is resting on the side with three dots, then the number of dots on the side at
the top is either 1 or 5.
5. Answer 4: From figures (i), (ii) and (iii), we conclude that 3, 4, 2 and 6 lie adjacent to 5. Therefore, 1 must lie opposite 5.
From figures (i), (iii) and (iv), we conclude that 4, 5, 6 and 1 lie adjacent to 3. Therefore, 2 must lie opposite 3. Now, we
have 1 opposite 5 and 2 opposite 3. Hence, 4 must lie opposite 6.

6. Answer 4: In fig. (X), one of the dots lies in the region common to the circle and the square only, another dot lies in the
region common to the square, the triangle and the rectangle only and the third dot lies in the region common to the triangle
and the rectangle only. In each of the figures (1), (2) and (3) there is no region common to the square, the triangle and the
rectangle only. Only fig. (4) consists of all the three types of regions.
7. Answer 2: when you fold the figure, 5 dots will appear.

8. Answer 1:
9. Answer 4:Water image

10. 2. The simplest triangles are BFG, CGH, EFM, FMG, GMN, GHN, HNI, LMK, MNK and KNJ i.e. 10 in number. The
triangles composed of three components each are FAK and HKD i.e. 2 in number. The triangles composed of four
components each are BEN, CMI, GLJ and FHK i.e. 4 in number. The triangles composed of eight components each are
BAJ and OLD i.e. 2 in number. Thus, there are 10 + 2 + 4 + 2 = 18 triangles in the given figure.

11. 3. The Horizontal lines are DF and BC i.e. 2 in number. The Vertical lines are DG, AH and FI i.e. 3 in number. The
Slanting lines are AB, AC, BF and DC i.e. 4 in number. Thus, there are 2 + 3 + 4 = 9 straight lines in the figure. Now, we
shall count the number of triangles in the figure. The simplest triangles are ADE, AEF, DEK, EFK, DJK, FLK, DJB, FLC,
BJG and LIC i.e. 10 in number. The triangles composed of two components each are ADF, AFK, DFK, ADK, DKB, FCK,
BKH, KHC, DGB and FIC i.e. 10 in number. The triangles composed of three components each are DFJ and DFL i.e. 2 in
number. The triangles composed of four components each are ABK, ACK, BFI, CDG, DFB, DFC and BKC i.e. 7 in
number. The triangles composed of six components each are ABH, ACH, ABF, ACD, BFC and CDB i.e. 6 in number.
There is only one triangle i.e. ABC composed of twelve components. There are 10 + 10 + 2 + 7 + 6+ 1 = 36 triangles in
the figure.
12. 5. The black leaf rotates 135oACW and the white leaf rotates 135 oCW.
13. 3. The line segment along one of the sides of the figure moves two spaces ACW (each space is equal to a side of the
figure) and the other element moves three spaces CW.

14. 3.

15. 2.
71 | P a g e For classes | Shortcut workshops | mocks | books Cetking –
09594441448 | 09930028086| 09820377380 | www.cetking.com
MBA CET 2024 Expected paper
16. A. 1, 4, 7 are all (two-dimensional) quadrilaterals. 2, 5, 8 are all three-dimensional figures. 3, 6, 9 are all (two-
dimensional) triangles.
17. Option B is correct when folded
18. Answer 3: Figure rotate by 90 deg CW. Dots and arrows decrease by 1
19. Answer 4: In each step, the CW-end element moves to the ACW- end position.
20. 2. In each row, the second figure is obtained from the first figure by adding two mutually perpendicular line segments at
the centre and the third figure is obtained from the first figure by adding four circles outside the main figure.

21. 3.
22. 2 is correct.
23. Answer 2: The CW end element moves to ACW side and new element is added on its ACW side
24. Answer 3: Elem interchange their positions once horizontally and vertically
25. Answer 3: In step 1, two elem on CW side interchange positions. ACW element move to centre and centre one is replaced.

Solutions
1. Ans.(b) Sol. GST Compensation Cess has been introduced to make good apprehended losses to
States in the
2. Ans.(c) Sol. as a subject vested with rights and duties. However, within the parameters of law, it
has never been confined to
3. Ans.(d) Sol. faces is that the groups perceived to be disadvantaged consist of a very large segment
of Indian society
4. Ans. (a) are widely accepted
5. Ans.(b) had been cheated

6. Ans.(c) Sol. how the thief made


7. Ans.(c) Sol. to make both ends meet
8. Ans.(b) Sol. had already been undetermined
9. Ans.(e) All are incorrect
10. Ans.(e) The United States said Russia was using Ukraine's biggest nuclear power plant as a
"nuclear shield" by stationing troops there.

11. D. Bacchanalian
12. A. Inoculation
13. D. Lackadaisical
14. C. Efflorescence
15. A. Judicious

16. Answer: Option D


17. Answer: Option C

18. Answer: Option E


19. Answer: Option B
20. Answer: Option C
21. Answer: Option A

22. Answer: Option C


23. Answer: Option D
24. Answer: Option C
25. Answer: Option A

1. Answer: Option D
Let the son's present age be x years. Then, man's present age = (x + 24) years.
(x + 24) + 2 = 2(x + 2)
72 | P a g e For classes | Shortcut workshops | mocks | books Cetking –
09594441448 | 09930028086| 09820377380 | www.cetking.com
MBA CET 2024 Expected paper
x + 26 = 2x + 4
x = 22.

2. Answer: Option B
Angle traced by hour hand in 17/2 hrs = 360/12 x 17/2 = 255°.
Angle traced by min. hand in 30 min. = 360/60 x 30° = 180°.
Required angle = (255 - 180)° = 75°.

3. Answer: Option B
For managing, A received = 5% of Rs. 7400 = Rs. 370.
Balance = Rs. (7400 - 370) = Rs. 7030.
Ratio of their investments = (6500 x 6) : (8400 x 5) : (10000 x 3)
= 39000 : 42000 : 30000
= 13 : 14 : 10
B's share = Rs. 7030 x 14/37 = Rs. 2660.

4. Answer: Option A

5. Answer: Option A

6. Answer: Option D
Let the speed of two trains be 7x and 8x km/hr.
Then, 8x = 400/4 = 100
x= 100 / 8= 12.5
Speed of first train = (7 x 12.5) km/hr = 87.5 km/hr.
7. Answer: Option C
The word 'LEADING' has 7 different letters.
When the vowels EAI are always together, they can be supposed to form one letter.
Then, we have to arrange the letters LNDG (EAI).
73 | P a g e For classes | Shortcut workshops | mocks | books Cetking –
09594441448 | 09930028086| 09820377380 | www.cetking.com
MBA CET 2024 Expected paper
Now, 5 (4 + 1 = 5) letters can be arranged in 5! = 120 ways.
The vowels (EAI) can be arranged among themselves in 3! = 6 ways.
Required number of ways = (120 x 6) = 720.

8. Answer: Option D
We have: (l - b) = 23 and 2(l + b) = 206 or (l + b) = 103.
Solving the two equations, we get: l = 63 and b = 40.
Area = (l x b) = (63 x 40) m2 = 2520 m2.

9. Answer: Option B
Let the ten's digit be x and unit's digit be y.
Then, (10x + y) - (10y + x) = 36
9(x - y) = 36
x - y = 4.

10. Answer: Option C


Let the shares of A, B, C and D be Rs. 5x, Rs. 2x, Rs. 4x and Rs. 3x respectively.
Then, 4x - 3x = 1000
x = 1000.
B's share = Rs. 2x = Rs. (2 x 1000) = Rs. 2000.

11. Answer: Option C


Suppose, first pipe alone takes x hours to fill the tank .
Then, second and third pipes will take (x -5) and (x - 9) hours respectively to fill the tank.
1/x + 1/(x - 5) = 1/(x - 9)
x - 5 + x / x(x - 5) = 1/(x - 9)
(2x - 5)(x - 9) = x(x - 5)
x2 - 18x + 45 = 0
(x - 15)(x - 3) = 0
x = 15. [neglecting x = 3]

12. Answer: Option B

13. Answer: Option C


A : B = 60 : 45.
A : C = 60 : 40.
B/C = B/A x A/c = 45 / 60 x 60/40 = 45 / 40 = 90/80
B can give C 10 points in a game of 90.
74 | P a g e For classes | Shortcut workshops | mocks | books Cetking –
09594441448 | 09930028086| 09820377380 | www.cetking.com
MBA CET 2024 Expected paper

14. Answer: Option C


In two throws of a dice, n(S) = (6 x 6) = 36.
Let E = event of getting a sum ={(3, 6), (4, 5), (5, 4), (6, 3)}.
P(E) =4/36 = 1/9

15. Answer: Option C

16. Answer: Option B


Explanation:
(a) Since loga a = 1, so log10 10 = 1.
(b) log (2 + 3) = log 5 and log (2 x 3) = log 6 = log 2 + log 3
log (2 + 3) log (2 x 3)
(c) Since loga 1 = 0, so log10 1 = 0.
(d) log (1 + 2 + 3) = log 6 = log (1 x 2 x 3) = log 1 + log 2 + log 3.
So, (b) is incorrect.

17. Answer: Option C


Man's rate in still water = (15 - 2.5) km/hr = 12.5 km/hr.
Man's rate against the current = (12.5 - 2.5) km/hr = 10 km/hr.

18. Answer: Option D


Let the speed of the boat in still water be x kmph. Then,
Speed downstream = (x + 3) kmph,
Speed upstream = (x - 3) kmph.
(x + 3) x 1 = (x - 3) x 3/2
2x + 6 = 3x - 9
x = 15 kmph.

19. Answer: Option A


Let the required weight be x kg.
Less weight, Less cost (Direct Proportion)
250 : 200 :: 60 : x => 250 x x = (200 x 60)
x= (200 x 60)/250
x = 48.

20. Answer: Option B


75 | P a g e For classes | Shortcut workshops | mocks | books Cetking –
09594441448 | 09930028086| 09820377380 | www.cetking.com
MBA CET 2024 Expected paper
Let the required number of revolutions made by larger wheel be x.
Then, More cogs, Less revolutions (Indirect Proportion)
14 : 6 :: 21 : x 14 x x = 6 x 21
x = 6 x 21
14
x = 9.

21. Answer: Option A

22. Answer: Option C


3x - y = 27 = 33 x - y = 3 ....(i)
3x + y = 243 = 35 x + y = 5 ....(ii)
On solving (i) and (ii), we get x = 4.

23. Answer: Option A


N = H.C.F. of (4665 - 1305), (6905 - 4665) and (6905 - 1305)
= H.C.F. of 3360, 2240 and 5600 = 1120.
Sum of digits in N = ( 1 + 1 + 2 + 0 ) = 4

24. Answer: Option A


Let the average age of the whole team by x years.
11x - (26 + 29) = 9(x -1)
11x - 9x = 46
2x = 46
x = 23.
So, average age of the team is 23 years.

25. Answer: Option C


On 31st December, 2005 it was Saturday.
Number of odd days from the year 2006 to the year 2009 = (1 + 1 + 2 + 1) = 5 days.
On 31st December 2009, it was Thursday.
Thus, on 1st Jan, 2010 it is Friday.

76 | P a g e For classes | Shortcut workshops | mocks | books Cetking –


09594441448 | 09930028086| 09820377380 | www.cetking.com

You might also like